PRACTICE TEST 2 ANSWER SHEET 91
ANSWER SHEET
ACT PRACTICE TEST 2
Answer Sheet
ENGLISH
1
A
B
C
D
2
F
G
H
J
3
A
B
C
D
4
F
G
H
J
5
A
B
C
D
6
F
G
H
J
7
A
B
C
D
8
F
G
H
J
9
A
B
C
D
10
F
G
H
J
11
A
B
C
D
12
F
G
H
J
13
A
B
C
D
14
F
G
H
J
15
A
B
C
D
16
F
G
H
J
17
A
B
C
D
18
F
G
H
J
19
A
B
C
D
20
F
G
H
J
21
A
B
C
D
22
F
G
H
J
23
A
B
C
D
24
F
G
H
J
25
A
B
C
D
26
F
G
H
J
27
A
B
C
D
28
F
G
H
J
29
A
B
C
D
30
F
G
H
J
31
A
B
C
D
32
F
G
H
J
33
A
B
C
D
34
F
G
H
J
35
A
B
C
D
36
F
G
H
J
37
A
B
C
D
38
F
G
H
J
39
A
B
C
D
40
F
G
H
J
41
A
B
C
D
42
F
G
H
J
43
A
B
C
D
44
F
G
H
J
45
A
B
C
D
46
F
G
H
J
47
A
B
C
D
48
F
G
H
J
49
A
B
C
D
50
F
G
H
J
51
A
B
C
D
52
F
G
H
J
53
A
B
C
D
54
F
G
H
J
55
A
B
C
D
56
F
G
H
J
57
A
B
C
D
58
F
G
H
J
59
A
B
C
D
60
F
G
H
J
61
A
B
C
D
62
F
G
H
J
63
A
B
C
D
64
F
G
H
J
65
A
B
C
D
66
F
G
H
J
67
A
B
C
D
68
F
G
H
J
69
A
B
C
D
70
F
G
H
J
71
A
B
C
D
72
F
G
H
J
73
A
B
C
D
74
F
G
H
J
75
A
B
C
D
MATHEMATICS
1
A
B
C
D
E
2
F
G
H
J
K
3
A
B
C
D
E
4
F
G
H
J
K
5
A
B
C
D
E
6
F
G
H
J
K
7
A
B
C
D
E
8
F
G
H
J
K
9
A
B
C
D
E
10
F
G
H
J
K
11
A
B
C
D
E
12
F
G
H
J
K
13
A
B
C
D
E
14
F
G
H
J
K
15
A
B
C
D
E
16
F
G
H
J
K
17
A
B
C
D
E
18
F
G
H
J
K
19
A
B
C
D
E
20
F
G
H
J
K
21
A
B
C
D
E
22
F
G
H
J
K
23
A
B
C
D
E
24
F
G
H
J
K
25
A
B
C
D
E
26
F
G
H
J
K
27
A
B
C
D
E
28
F
G
H
J
K
29
A
B
C
D
E
30
F
G
H
J
K
31
A
B
C
D
E
32
F
G
H
J
K
33
A
B
C
D
E
34
F
G
H
J
K
35
A
B
C
D
E
36
F
G
H
J
K
37
A
B
C
D
E
38
F
G
H
J
K
39
A
B
C
D
E
40
F
G
H
J
K
41
A
B
C
D
E
42
F
G
H
J
K
43
A
B
C
D
E
44
F
G
H
J
K
45
A
B
C
D
E
46
F
G
H
J
K
47
A
B
C
D
E
48
F
G
H
J
K
49
A
B
C
D
E
50
F
G
H
J
K
51
A
B
C
D
E
52
F
G
H
J
K
53
A
B
C
D
E
54
F
G
H
J
K
55
A
B
C
D
E
56
F
G
H
J
K
57
A
B
C
D
E
58
F
G
H
J
K
59
A
B
C
D
E
60
F
G
H
J
K
92 PRACTICE TEST 2 ANSWER SHEET
READING
1
A
B
C
D
2
F
G
H
J
3
A
B
C
D
4
F
G
H
J
5
A
B
C
D
6
F
G
H
J
7
A
B
C
D
8
F
G
H
J
9
A
B
C
D
10
F
G
H
J
11
A
B
C
D
12
F
G
H
J
13
A
B
C
D
14
F
G
H
J
15
A
B
C
D
16
F
G
H
J
17
A
B
C
D
18
F
G
H
J
19
A
B
C
D
20
F
G
H
J
21
A
B
C
D
22
F
G
H
J
23
A
B
C
D
24
F
G
H
J
25
A
B
C
D
26
F
G
H
J
27
A
B
C
D
28
F
G
H
J
29
A
B
C
D
30
F
G
H
J
31
A
B
C
D
32
F
G
H
J
33
A
B
C
D
34
F
G
H
J
35
A
B
C
D
36
F
G
H
J
37
A
B
C
D
38
F
G
H
J
39
A
B
C
D
40
F
G
H
J
SCIENCE
1
A
B
C
D
2
F
G
H
J
3
A
B
C
D
4
F
G
H
J
5
A
B
C
D
6
F
G
H
J
7
A
B
C
D
8
F
G
H
J
9
A
B
C
D
10
F
G
H
J
11
A
B
C
D
12
F
G
H
J
13
A
B
C
D
14
F
G
H
J
15
A
B
C
D
16
F
G
H
J
17
A
B
C
D
18
F
G
H
J
19
A
B
C
D
20
F
G
H
J
21
A
B
C
D
22
F
G
H
J
23
A
B
C
D
24
F
G
H
J
25
A
B
C
D
26
F
G
H
J
27
A
B
C
D
28
F
G
H
J
29
A
B
C
D
30
F
G
H
J
31
A
B
C
D
32
F
G
H
J
33
A
B
C
D
34
F
G
H
J
35
A
B
C
D
36
F
G
H
J
37
A
B
C
D
38
F
G
H
J
39
A
B
C
D
40
F
G
H
J
RAW SCORES SCALE SCORES
DATE TAKEN:
ENGLISH
ENGLISH
MATHEMATICS MATHEMATICS ENGLISH/WRITING
READING READING
SCIENCE SCIENCE
COMPOSITE SCORE
Refer to the Scoring Worksheet on page 150 for help in determining your Raw and Scale Scores.
Begin WRITING TEST here.
If you need more space, please continue on the next page.
1
Do not write in this shaded area.
You may wish to remove these sample answer document pages to respond to the practice ACT Writing Test.
Cut Here
WRITING TEST
If you need more space, please continue on the back of this page.
2
Do not write in this shaded area.
WRITING TEST
If you need more space, please continue on the next page.
3
PLEASE DO NOT WRITE IN THIS AREA.
Cut Here
WRITING TEST
STOP here with the Writing Test.
4
Do not write in this shaded area.
PRACTICE TEST 2 ENGLISH TEST 97
1 1
ENGLISH TEST
45 Minutes—75 Questions
DIRECTIONS: In the passages that follow, some words
and phrases are underlined and numbered. In the
answer column, you will find alternatives for the words
and phrases that are underlined. Choose the alternative
that you think is best, and fill in the corresponding
bubble on your answer sheet. If you think that the
original version is best, choose “NO CHANGE,” which
will always be either answer choice A or F. You will
also find questions about a particular section of the
passage, or about the entire passage. These questions
will be identified by either an underlined portion or by
a number in a box. Look for the answer that clearly
expresses the idea, is consistent with the style and tone
of the passage, and makes the correct use of standard
written English. Read the passage through once before
answering the questions. For some questions, you
should read beyond the indicated portion before you
answer.
PASSAGE I
A Blessing in Disguise
1. A. NO CHANGE
B. will be
C. was
D. have been
2. F. NO CHANGE
G. couple that
H. couple, Jean and Christine
J. couple, in a few hours
3. A. NO CHANGE
B. pouring from my mouth, speaking
C. pouring from my mouth speaking
D. pouring from my mouth by speaking
4. F. NO CHANGE
G. was eating
H. began to eat
J. would eat
Last spring, I had been
1
fortunate to be chosen to
participate in an exchange study program. In my
application essay, I was careful to express how much I
wanted to see France. I suppose my excitement really
came through in my words. Once I knew that I was going,
all I could think about was the fun of foreign travel and
making all sorts of new and interesting friends. While
traveling was inspiring and meeting people was exciting,
nothing about my semester in France was what I expected.
The moment I arrived in Paris, I was greeted by a nice
French couple who
2
would become my host parents. The
bit of French I had taken in high school began
pouring from my mouth. Speaking
3
the language would
only become more natural over the course of the semester.
At the airport, we all got into the couple’s little two-door
hatchback and began the journey to their townhouse in the
suburbs. We talked the whole way there, getting to know
one another bit by bit. Everyday thereafter, I eat
4
breakfast
GO ON TO THE NEXT PAGE.
98 PRACTICE TEST 2 ENGLISH TEST
1 1
5. A. NO CHANGE
B. since
C. therefore
D. then
6. F. NO CHANGE
G. host parent’s
H. host parents’
J. host’s parents
7. A. NO CHANGE
B. weeks to tend
C. weeks, tended
D. weeks
8. F. NO CHANGE
G. which would probably be about the same size.
H. which I hope would be closer to the supermarket.
J. OMIT the underlined portion and end the sentence
with a period.
9. A. NO CHANGE
B. English speaker and one who was not.
C. English speaker or a person, not an English speaker.
D. English speaker.
10. F. NO CHANGE
G. which
H. that
J. he who
11. Given that all the choices are true, which one pro-
vides the most relevant information with regard to the
narrators friendship with Paolo?
A. NO CHANGE
B. He hadn’t heard of a lot of my CDs, though.
C. We didn’t have a lot of classes together, but at least
we liked the same music.
D. I didn’t speak Portuguese, so it took some time to
start to understand each other.
12. F. NO CHANGE
G. talk about my Brazilian friend Paolo
H. talk about my Brazilian friend, Paolo,
J. talk, about my Brazilian friend Paolo,
with the two of them, so
5
we’d all go our separate ways for
the day. In the evening, my host mother would make
delicious dinners for the three of us. My entire experience
was joyous and exhilarating until I received some shocking
news from my program coordinator: there had been a
death in my host parents
6
extended family. They had to
travel outside France for several weeks, so tending
7
to all
the business that arises from an unforeseen death. That
afternoon, I had to move out of one family’s house and
into another so I’d have to repack my suitcases.
8
The exchange coordinator told me I’d have a roommate
this time and asked whether I could share a bedroom with
an English speaker or someone who didn’t speak English.
9
To avoid the temptation to speak my native language,
I asked not to be placed with an English-speaking
roommate. When I got to my new room, I introduced
myself to my new roommate Paolo, a Brazilian the same
age as I, whom
10
I was surprised to find playing one of my
favorite CDs on the stereo!
In just a few hours, we knew we’d be
11
attached at the hip for the rest of the term.
11
I left France with many stories, so when people ask me
what my favorite part of the trip was, they are always
surprised to hear me talk, about my Brazilian friend Paolo,
12
and the scores of weekdays in class, weeknights on the
town, and weekends exploring France we enjoyed together.
GO ON TO THE NEXT PAGE.
PRACTICE TEST 2 ENGLISH TEST 99
1 1
13. A. NO CHANGE
B. I love how people seem so different and are so
similar.
C. People seem so different, so I love how they end
up being so similar.
D. I love how people can seem so different, but end
up being so similar.
14. Which of the choices would be most appropriate here?
F. NO CHANGE
G. enjoyable
H. fun
J. supportive
15. Which of the following sentences, if inserted here,
would best conclude the essay as well as maintain the
positive tone established earlier in the essay?
A. France is an interesting place once you grasp the
language.
B. I would recommend an exchange program to any-
one who wants to experience foreign cultures.
C. High school is going to be quite boring now,
especially since my new friend Paolo won’t be
there.
D. It will be nice to graduate at the end of this year.
I love people, how they end up being so similar, but are
13
so different.
13
The most valuable
14
lesson I gained from
studying in France wasn’t just to respect the French people
but to respect all people, for your next best friend could be
just a continent away.
15
PASSAGE II
My Favorite Lunch Spot
16. F. NO CHANGE
G. apartment I’m renting
H. apartment I’m renting,
J. apartment, I’m renting
17. A. NO CHANGE
B. was located
C. had been
D. will be
18. F. NO CHANGE
G. fountain in
H. fountain in,
J. fountain; in
A few blocks south of the apartment, I’m renting,
16
Joe’s
Lunch Bucket serves up amazing sandwiches. The owner
runs the place, so he stays open as late as he has
customers, usually until some time after midnight. The
restaurant is
17
at the end of an alley, and if you sit on the last
stool by the window, you can see the big public
fountain, in
18
the adjacent square. There are usually swarms
of children and teenagers milling around the area; no one
really enforces the curfew, especially in the summer when
the nights are warm and families stroll around the shops
and public spaces downtown.
[1] Joe has a menu stuck to the front window with
masking tape that is yellowed and cracked from years in
the sun. [2] Never mind the dingy interior, noisy kitchen,
and lack of parking. [3] I just go there for the food.
GO ON TO THE NEXT PAGE.
100 PRACTICE TEST 2 ENGLISH TEST
1 1
19. A. NO CHANGE
B. sinfully delicious
C. sinful deliciousness
D. sinful delicious
20. F. NO CHANGE
G. Newcomers to Joe’s
H. Newcomers to Joe’s who need a menu to order
J. People who’ve never had the pleasure of a Joe’s
sandwich
21. A. NO CHANGE
B. sauerkraut, spilling
C. sauerkraut, spilling,
D. sauerkraut spilling
22. For the sake of logic and coherence, Sentence 4 of this
paragraph should be placed:
F. where it is now.
G. before Sentence 1.
H. after Sentence 2.
J. after Sentence 1.
23. Which choice most effectively guides the reader from
the preceding paragraph into this new paragraph?
A. NO CHANGE
B. Joe takes good care of his property.
C. May be Joe learned his sandwich secrets at culinary
school.
D. Good food is the key to Joe’s success.
24. F. NO CHANGE
G. belies that the
H. belies, and the
J. belies the
25. A. NO CHANGE
B. clicks to whine
C. clicking and whining
D. click and whine
26. At this point, the writer wants to add a sentence that
would further describe the condition of the restau-
rant. Which of the following sentences would best
accomplish this?
F. Regardless of the appearance of the place, I still
enjoy my delicious sandwich.
G. I would like to see who his maintenance man is.
H. If I had a restaurant, I’d make sure it was clean.
J. People seem to ignore the building, though.
[4] I’ve never stopped to read it and, as far as I can tell,
neither have the other regulars. [5] I like to sit at the bar
along the window and relax with the sinfully deliciousness
19
of Joe’s Special Rueben. [6] Newcomers to Joe’s who
20
have never seen his creations
20
marvel at the stack of
corned beef and sauerkraut; spilling
21
from the bread onto
my paper plate.
22
Joe’s is my home away from home.
23
The sign outside is
hardly eye-catching and the restaurant always appears to
be dimly lit, but one can’t help noticing the large smiley
face decal affixed to the front door that reads “Keep
Smiling!” The sandwiches certainly make me smile, but I
can’t say they do the same for Joe himself. His constant
ugly expression belies, the
24
care that he takes with his
meats, breads, and cheeses. So, too, does his quirky
restaurant. The counters are dented and scratched from
years of knife abuse. The old refrigerator case
clicks and whines
25
constantly. As I savor my sandwich, my
gaze always drifts toward the caulk along the window
panes, once white, which is slowly deteriorating with the
rest of the place. In fact, I’ve often thought to offer Joe my
painting services in exchange for some sustenance.
26
GO ON TO THE NEXT PAGE.
PRACTICE TEST 2 ENGLISH TEST 101
1 1
27. A. NO CHANGE
B. many
C. mostly
D. none
28. The writer is considering deleting the following phrase
from the preceding sentence:
chicken salad, corned beef, and the like.
If the writer were to make this deletion, the essay would
primarily lose:
F. foreshadowing of the conclusion.
G. irrelevant information.
H. specific descriptive material.
J. an understatement of important information.
29. A. NO CHANGE
B. When I am back home and reminiscing, I picture
all of these people with their favorite meals.
C. Back home, I picture all these people with the
favorite meals when I reminisce.
D. Reminiscing back home makes me picture all these
people with their favorite meals.
30. F. NO CHANGE
G. Bucket, and
H. Bucket and
J. Bucket;
The food is, after all, the only charm this little place needs.
A lot of people pay daily visits to the sandwich shop.
I know much
27
of their faces by now, but I could more
easily recall their tastes in sandwiches. Older people like
the classics—chicken salad, corned beef, and the like.
28
Kids come in after school for grilled cheeses or Joe’s
tuna salad. Back home, as I am reminiscing on this place,
29
I picture all these people with their favorite meals.
29
Perhaps it’s the familiarity that makes Joe’s my favorite
sandwich shop. I know that I can come in whenever I
please and someone would look away from a savory
sandwich and offer a friendly hello. It’s nice to know that
Joe’s Lunch Bucket. And
30
its neighborly ambience are just
a short walk away.
PASSAGE III
Slowly Spanning the Straits
31. A. NO CHANGE
B. knew
C. have known
D. knows
32. F. NO CHANGE
G. eighteenth, and nineteenth
H. eighteenth and nineteenth
J. eighteenth and nineteenth,
33. A. NO CHANGE
B. proved
C. proves
D. have proven
The Straits of Mackinac, located between Lake Huron
and Lake Michigan, divide Michigan’s Upper and Lower
Peninsulas. Native Americans in the former wilderness
territory know
31
how to paddle between several islands to
make their way across the Straits. Settlers in the
eighteenth, and nineteenth,
32
centuries crossed the Straits by
ferry. However, ferries soon prove
33
to be costly in both lives
GO ON TO THE NEXT PAGE.
102 PRACTICE TEST 2 ENGLISH TEST
1 1
34. F. NO CHANGE
G. Giving up by the 1880s
H. Until the 1880s
J. In terms of the 1880s
35. The writer is considering deleting the underlined por-
tion from the sentence. If the writer were to delete this
phrase, the essay would primarily lose:
A. a minor detail in the essay’s opening paragraph.
B. an explanation of the impetus for discussion of a
potential bridge.
C. the writers opinion about the historical signifi-
cance of the Mackinac Bridge.
D. an indication of Michigan’s desire to keep pace
with the transportation development taking place in
New York City.
36. F. NO CHANGE
G. for the project
H. by the project
J. of the project
37. A. NO CHANGE
B. peninsulas so
C. peninsulas, but
D. peninsulas; and
38. F. NO CHANGE
G. their great cost
H. it’s great cost
J. its great cost
39. A. NO CHANGE
B. however that
C. however
D. however,
and money. By the 1880s
34
, the Michigan Legislature had
begun discussing the idea of building a bridge to span the,
Strait noting the success of the newly-built Brooklyn
35
Bridge
35
. However, many hurdles stood in the way.
During the late nineteenth century, the Legislature heard
plans for an elaborate system of bridges and causeways
that would use three islands as intermediate points.
However, no action was ever taken on the project
36
. In the
1920s, an assembly ordered resumption of ferry service
between the peninsulas; so
37
within five years, Governor
Fred Green felt there great cost
38
warranted investigation of
the bridge idea once again. The State Highway Authority
concluded that a bridge could be built for around
$30 million.
In the 1930s, The Mackinac Bridge Authority twice
sought federal funding for construction of the bridge, but
was denied each time. Even so, a route was plotted and
careful study of the lakebed and the rock below began.
Any progress, however, that
39
was put on hold for the
duration of World War II, and it was not until 1950 that
funds were fully invested in the bridge project.
Construction of the Mackinac Bridge finally began in
1954. It would become a crowning achievement for design
engineer David Steinman and, for years, would be
GO ON TO THE NEXT PAGE.
PRACTICE TEST 2 ENGLISH TEST 103
1 1
40. If the writer were to delete the preceding sentence, the
paragraph would primarily lose:
F. an explanation of how the Mackinac Bridge was
erected.
G. details about the significance of the Mackinac
Bridge.
H. background information on the history of building
bridges.
J. biographical information about David Steinman.
41. A. NO CHANGE
B. On November 1, 1957, the Mackinac Bridge
opened, in spite of decades of problems, to traffic.
C. The Mackinac Bridge opened to traffic, in spite of
decades of problems, on November 1, 1957.
D. In spite of decades of problems, the Mackinac
Bridge opened to traffic on November 1, 1957.
42. F. NO CHANGE
G. Since,
H. Meanwhile,
J. Historically,
43. A. NO CHANGE
B. highway drivers and travelers
C. drivers—and highway travelers—
D. highway travelers
44. Which of the following alternatives to the underlined
portion would be LEAST acceptable in terms of the
context of this sentence?
F. mark the union of Michigan’s two peninsulas.
G. serve as a symbol of suspension bridges around the
world.
H. provide an image of strength and grace to all who
cross it.
J. pay tribute to the progress of a great state.
Question 45 asks about the preceding passage as a whole.
45. Suppose the writer had intended to write a brief essay
that describes the entire process of designing and build-
ing the Mackinac Bridge. Would this essay successfully
fulfill the writers goal?
A. Yes, because it offers such details as the material
of the superstructure, the identity of the designer,
and the cost of construction.
B. Yes, because it explains in detail each step in the
design and construction of the bridge.
C. No, because it focuses primarily on the difficulty
and delay in seeing construction of a bridge across
the Strait come to fruition.
D. No, because it is primarily a historical essay about
the motivation behind the bridge project.
the longest suspension bridge in the world.
40 U.S. Steel
Company received the contract to build the massive steel
superstructure. It was a two-and-a-half year ordeal that
cost the state more than $44 million and cost five men
their lives. On November 1, 1957, the Mackinac Bridge,
41
in spite of decades of problems, opened to traffic.
41
Those
who did not know the history of the project were elated by
the bridge’s “on schedule” completion.
Today,
42
the Mackinac Bridge is as solid as ever. In 1998
it collected its 100 millionth toll. It will continue to serve
drivers and highway travelers
43
well into the future and
stand as a monument to Michigan’s perseverance.
44
GO ON TO THE NEXT PAGE.
104 PRACTICE TEST 2 ENGLISH TEST
1 1
PASSAGE IV
The following paragraphs may or may not be in the most
logical order. You may be asked questions about the log-
ical order of the paragraphs, as well as where to place
sentences logically within any given paragraph.
Care with Cards
[1]
46. F. NO CHANGE
G. We decided to go to a department store instead, to
look for shoes.
H. (We were in the mall to do some shoe shopping.)
J. OMIT the underlined portion.
47. A. NO CHANGE
B. whom I knew
C. I knew
D. OMIT the underlined portion.
48. F. NO CHANGE
G. room, brimming
H. room brimming
J. room brimming,
49. A. NO CHANGE
B. has
C. was
D. could of been
50. Which choice would most effectively and appropriately
lead the reader from the topic of Paragraph 1 to that of
Paragraph 2?
F. NO CHANGE
G. Electronic forms of entertainment involve the cre-
ative mind instead of the organized, mathematical
mind.
H. I find nothing entertaining about television, video
games, and other electronic entertainment.
J. Baseball cards decreased in popularity while elec-
tronic entertainment has increased.
Does anyone have a real hobby anymore? I must admit
I was disheartened when my brother, younger by 10 years,
didn’t want to go into the sports card shop with me.
We went to a department store instead.
46
Every boy
which I have known
47
as a kid had a box in his
room, brimming,
48
with cards. Some boys were into
basketball and football cards, but my passion was for
baseball cards. I couldn’t believe the shocked look on my
brothers face when he saw some of the expensive
offerings in the shop’s window display. He just couldn’t
appreciate the history behind the cards and the care taken
to preserve them over the decades. For him, no piece of
cardboard are
49
worth any sum of money. He would rather
have a video screen to distract him.
[2]
[1] I appreciate the arguments in favor of television,
50
video games, and other electronic entertainment.
50
GO ON TO THE NEXT PAGE.
PRACTICE TEST 2 ENGLISH TEST 105
1 1
51. Which of the following alternatives to the underlined
portion would be LEAST acceptable?
A. and, generally,
B. but, thoroughly
C. and, therefore,
D. and, as such,
52. For the sake of logic and coherence of Paragraph 2 this
sentence should be:
F. placed where it is now.
G. placed after Sentence 1.
H. placed after Sentence 7.
J. OMITTED, because the paragraph focuses only on
electronic forms of entertainment.
53. A. NO CHANGE
B. alike.
C. alike, which can benefit both groups.
D. alike that want a wholesome hobby.
54. F. NO CHANGE
G. each and every
H. every
J. every unique
55. A. NO CHANGE
B. we often shared
C. I often share
D. I then shared
56. F. NO CHANGE
G. too, encouraging
H. too; encouraging
J. too, by encouraging
57. A. NO CHANGE
B. at the screen.
C. for the screen.
D. OMIT the underlined portion and end the sentence
with a period.
[2] The technology is dynamic and, for the most part,
51
engaging. [3] There’s indeed something for everyone.
[4] I also believe, however, that these new forms of
entertainment have taken time away from “unplugged”
fun. [5] A good hands-on hobby should be an important
52
part of any childhood.
52
[6] Instead of simple story books,
toddlers have interactive learning computers that read for
them. [7] When children aren’t watching satellite
television, they have console games to entertain them.
[8] It seems to me that more and more of the joys of
childhood are being lost to the allure of the video screen.
[3]
Building a sports card collection is a rewarding endeavor
for children and adults alike that is fun for everyone.
53
It
teaches quality lessons, such as patience and organization.
For young fingers, it develops a careful touch. I remember
how hard I tried to slide each card into a plastic sleeve
without damaging the delicate corners of the card. Nearly
all
54
weekend, there was a card show at the local mall where
I learned to bargain and trade for all the cards that I
needed to fill the gaps in my collection. Although I very
much enjoyed spending time alone looking at the cards,
I often shared
55
the experience with others. Card collecting
is a social activity too encouraging
56
the old and young to
swap cards and stories. Today’s electronic entertainment,
however, keeps people apart and does little to benefit
developing minds. The video screen silences spectators as
it holds their attention to the screen.
57
Television and video
games deliver instant gratification. Tune a channel or insert
a disc and off you go. Hours and hours of sedentary
GO ON TO THE NEXT PAGE.
106 PRACTICE TEST 2 ENGLISH TEST
1 1
58. F. NO CHANGE
G. your
H. one’s
J. people’s
59. A. NO CHANGE
B. instants
C. instantly
D. more instant
Question 60 asks about the preceding passage as a whole.
60. Suppose the writer had chosen to write an essay that
indicates that sports card collecting is superior to
electronic entertainment. Would this essay fulfill the
writers goal?
F. No, because the writer admits that electronic enter-
tainment has become more popular than sports card
collecting.
G. No, because the writer states that electronic enter-
tainment is dynamic and engaging.
H. Yes, because the writer claims that, unlike elec-
tronic entertainment, sports card collecting teaches
valuable life skills such as organization and care-
ful handling of fragile items, and also provides a
medium for social interaction.
J. Yes, because the writer suggests that any hands-on
hobby is better than watching television.
satisfaction are at their
58
fingertips!
[4]
I am concerned that this trend toward electronics will
lead to less physical activity and make the fun in life
effortless and instant
59
available. The younger generation
needs to know that pleasure can also come from a hobby
that demands patience, care, hard work, and concentration.
PASSAGE V
The following paragraphs may or may not be in the most
logical order. You may be asked questions about the log-
ical order of the paragraphs, as well as where to place
sentences logically within any given paragraph.
Spies Online
61. A. NO CHANGE
B. wait, until,
C. wait until,
D. wait until
[1]
People who choose to use a personal computer to
connect to the Internet should know the risks that this
poses. Most computer users have some experience with
slow computers, unexplainable program crashes, and
indecipherable warnings about missing system files. These
same computer users are more likely to wait, until
61
GO ON TO THE NEXT PAGE.
PRACTICE TEST 2 ENGLISH TEST 107
1 1
62. F. NO CHANGE
G. spyware, which crippling
H. spyware, and it’s crippled by
J. spyware, and its crippled
63. A. NO CHANGE
B. As with
C. Unlike
D. Comparable to
64. F. NO CHANGE
G. except
H. always
J. instead of
65. A. NO CHANGE
B. spyware, however,
C. spyware, however
D. spyware however
66. Which choice is the most effective first sentence of
Paragraph 3?
F. NO CHANGE
G. Among the most serious spyware programs are
those called “keystroke loggers.”
H. Most people don’t know their computers are
infected with spyware.
J. Due to unsecured internet connections, spyware is
far more prevalent than computer viruses.
67. A. NO CHANGE
B. initiate depletion of system resources,
C. lead to depleting system resources,
D. deplete system resources,
68. F. NO CHANGE
G. Detection utilities
H. Spyware can be found by detection utilities that
J. Detection utilities that find spyware
these problems get too bad to manage. They would buy a
new system entirely before trying to fix their current one.
Online, the biggest threat is spyware, which is crippling
62
unsecured computers and data networks around the world.
[2]
Like
63
a computer virus, a spyware program is not purely
malicious. The developer of the spyware program stands to
gain from installing it on your computer, often
64
just in
information, but usually financially, too. These programs
may monitor your online activity and track your
keystrokes and buying habits. This data is sold to
marketing agencies for demographic research, and to more
unscrupulous firms that will bombard you with email
solicitations and sales calls. Not all spyware; however
65
has
legitimate commerce behind it.
[3]
Both computer viruses and spyware can cause problems.
66
Other programs show up as system messages, luring
unaware users to click their way into corrupting their own
operating systems and revealing sensitive personal
information. Not only do these programs cause a
67
depletion of system resources,
67
but they waste time and test
the nerves of even the most patient user.
[4]
There are plenty of solutions designed to eliminate the
spyware problem. The first step is to rid your computer of
any unwanted programs. Detection utilities that detect
68
spyware
68
are widely available, many at no cost,
GO ON TO THE NEXT PAGE.
108 PRACTICE TEST 2 ENGLISH TEST
1 1
69. A. NO CHANGE
B. that scan the computer for undesirable programs
and remove them.
C. that scan the computer for undesirable programs
and removes them.
D. that scans the computer for undesirable programs
then removing them.
70. F. NO CHANGE
G. has been protected
H. protects them
J. is protected
71. In this paragraph, the writer intends to recommend a
number of sound web surfing habits. This is to be the
second recommendation. Given that all of the choices
are true, which one would best accomplish the writers
intention?
A. NO CHANGE
B. Don’t buy anything online from a store with no
physical address.
C. Shut down your computer when you aren’t using it.
D. Know the various names of spyware programs.
72. The best placement for the underlined portion would
be:
F. where it is now.
G. after the word Go.
H. after the word firewall.
J. at the beginning of the sentence.
73. A. NO CHANGE
B. it
C. it, while it
D. it, it,
Questions 74 and 75 ask about the preceding passage as
a whole.
74. Upon reviewing this essay and realizing that some
information has been left out, the writer composes
the following sentence, incorporating that missing
information:
If you own a computer, it is vital to understand
it, for noxious software is becoming increasingly
sophisticated and infectious.
The most logical and effective place to add this
sentence would be after the last sentence of Paragraph:
F. 2.
G. 3.
H. 4.
J. 5.
that scans the computer for undesirable programs and
69
then removes them.
69
Once this is accomplished, the utilities
monitor the system constantly to prevent any new
installation of spyware. It is important to understand how
your computer protects
70
and to keep your software updated.
[5]
Good web surfing habits are essential, too. Avoid web
sites you don’t trust. Spyware originates from many
71
kinds of web sites.
71
Go online never
72
without a firewall and
active virus and spyware protection. Remember that a
computer is just a machine. If you turn it on and never
touch it, it
73
will likely remain fast and reliable. It is
generally what the user does to the computer that affects it.
GO ON TO THE NEXT PAGE.
PRACTICE TEST 2 ENGLISH TEST 109
1 1
75. Suppose the writer had decided to write an essay
discussing the moral and ethical consequences of pro-
gramming spyware to illicitly collect private informa-
tion. Would this essay successfully fulfill the writers
goal?
A. Yes, because the essay explains the moral and eth-
ical consequences when spyware is installed on a
computer.
B. Yes, because the essay details the process of rid-
ding a computer of spyware, which helps the reader
to understand the consequences of programming
spyware.
C. No, because the essay does not explain how to
program spyware, so the reader has no basis for
making a moral or ethical judgment.
D. No, because the essay limits itself to a brief descrip-
tion of spyware and the basic precautions to be
taken against it.
END OF THE ENGLISH TEST.
STOP! IF YOU HAVE TIME LEFT OVER, CHECK YOUR WORK ON THIS SECTION ONLY.
110 PRACTICE TEST 2 MATHEMATICS TEST
2 2
MATHEMATICS TEST
60 Minutes—60 Questions
DIRECTIONS: Solve each of the problems in the time
allowed, then fill in the corresponding bubble on your
answer sheet. Do not spend too much time on any one
problem; skip the more difficult problems and go back
to them later.
You may use a calculator on this test. For this test
you should assume that figures are NOT necessarily
drawn to scale, that all geometric figures lie in a
plane, and that the word
line is used to indicate a
straight line.
DO YOUR FIGURING HERE.
1. In the standard (x,y) coordinate plane, point X has coor-
dinates (4,0) and point Y has coordinates (0,8). What
are the coordinates of the midpoint of
XY?
A. (6,1)
B. (2,4)
C. (0,2)
D. (2,4)
E. (6,1)
2. Given right triangle MNO below, how many units long
is
NO?
F. 2
2
G. 4
H. 6
J.
60
K. 8
3. A distance in meters, M, can be approximated by mul-
tiplying a distance in yards, Y, by 1.0936. Which of the
following expresses this approximation method? (Note:
The symbol means “is approximately equal to.”)
A. M
Y
1.0936
B. M
1.0936
Y
C. M Y(1.0936)
D. M Y + 1.0936
E. M Y(1.0936Y)
GO ON TO THE NEXT PAGE.
PRACTICE TEST 2 MATHEMATICS TEST 111
2 2
DO YOUR FIGURING HERE.
4. Seth has 4 plaid shirts and 5 solid-colored shirts hang-
ing together in a closet. In his haste to get ready for
work, he randomly grabs 1 of these 9 shirts. What is the
probability that the shirt Seth grabs is plaid?
F.
1
5
G.
1
4
H.
4
9
J.
1
9
K.
4
5
5. The daily totals of enrollments at Sunnyside Summer
Camp last Monday through Saturday were 17, 19, 23,
14, 25, and 28. What was the average number of
enrollments per day?
A. 126
B. 28
C. 21
D. 18
E. 14
6. In the figure showing PQR below, line l is parallel
to line m. Which one of the following angles must be
congruent to y?
F. 1
G. 2
H. 3
J. 4
K. 5
7. A carton of paper is priced at $27.00 now. If the paper
goes on sale for 25% off the current price, what will be
the sale price of the carton?
A. $6.75
B. $20.25
C. $22.00
D. $26.75
E. $33.75
GO ON TO THE NEXT PAGE.
112 PRACTICE TEST 2 MATHEMATICS TEST
2 2
DO YOUR FIGURING HERE.
8. What is the slope of any line parallel to the line
2x 3y = 7?
F. 3
G.
2
3
H.
2
3
J. 2
K. 3
9. Andrew won a cash prize on a game show. Andrew paid
taxes of 30% on the original cash prize and had $28,000
remaining. How much was the original cash prize?
A. $19,600
B. $28,300
C. $36,400
D. $40,000
E. $84,000
10. Melissa had 3 fewer apples than Marcia. Then, she gave
2 apples to Marcia. Now how many fewer apples does
Melissa have than Marcia?
F. 0
G. 2
H. 3
J. 5
K. 7
11. What is the value of |5 a| if a = 9?
A. 14
B. 4
C. 4
D. 9
E. 14
12. For all m and n, (3m + n)(m
2
n) = ?
F. 3m
3
+ 2m
2
2n
G. m
3
2n
2
H. 2m
2
n n
2
J. 3m
2
+ 3mn 2n
2
K. 3m
3
3mn + m
2
n n
2
13. For all x, 13 2(x + 5) = ?
A. 2x + 3
B. 11x + 55
C. 13 + 10x
D. 23 2x
E. 23 + 2x
14. (n
7
)
11
is equivalent to:
F. n
77
G. n
18
H. 11n
4
J. 11n
7
K. 77n
GO ON TO THE NEXT PAGE.
PRACTICE TEST 2 MATHEMATICS TEST 113
2 2
DO YOUR FIGURING HERE.
15. What is the 217th digit after the decimal point in the
repeating decimal 0.
3456?
A. 0
B. 3
C. 4
D. 5
E. 6
16. The perimeter of a square is 48 centimeters. What is its
area, in square centimeters?
F. 12
G. 96
H. 144
J. 192
K. 2,304
17. What is the product of the 2 solutions of the equation
x
2
+ 3x 21 = 0?
A. 63
B. 21
C. 20
D. 20
E. 21
18. Which of the following expressions is a polynomial
factor of a
16
16?
F. a
4
4
G. a
4
+ 4
H. a
4
+ 2
J. a + 2
K. a 2
19. When n =
1
4
, what is the value of
2n 5
n
?
A. 18
B. 9
C. 3
D. 9
E. 18
20. A proofreader can read 40 pages in one hour. How many
pages can this proofreader read in 90 minutes?
F. 45
G. 60
H. 150
J. 360
K. 940
GO ON TO THE NEXT PAGE.
114 PRACTICE TEST 2 MATHEMATICS TEST
2 2
DO YOUR FIGURING HERE.
21. The area of a parallelogram may be found by multiplying
the base by the height. What is the area, in square inches,
of the parallelogram below?
A. 27
B. 36
C. 45
D. 48
E. 81
22. For a certain quadratic equation, ax
2
+ bx + c = 0,
the 2 solutions are x =
3
4
and x =
2
5
. Which of the
following could be factors of ax
2
+ bx + c?
F. (4x 3) AND (5x + 2)
G. (4x 2) AND (5x + 3)
H. (4x + 2) AND (5x 3)
J. (4x + 3) AND (5x 2)
K. (4x + 3) AND (5x + 2)
23. All sides of a rhombus are the same length, as shown
below.
If one diagonal is 12 inches long and the other is
32 inches long, how many inches long, to the nearest
hundredth of an inch, is a side of the rhombus?
A. 8.54
B. 17.09
C. 34.17
D. 35.78
E. 48.00
24. A rectangular parking lot that is 3 feet longer than it is
wide has an area of 550 square feet. How many feet
long is the parking lot?
F. 19
G. 20
H. 22
J. 25
K. 28
GO ON TO THE NEXT PAGE.
PRACTICE TEST 2 MATHEMATICS TEST 115
2 2
DO YOUR FIGURING HERE.
25. In the standard (x, y) coordinate plane, what is the slope
of the line joining the points (3,7) and (4,8)?
A. 15
B. 1
C.
1
7
D.
21
32
E. 15
26. Which of the following is the solution set of x+2>4?
F. {x: x < 6}
G. {x: x > 6}
H. {x: x < 2}
J. {x: x > 2}
K. {x: x < 6}
27. What is the center of the circle with equation (x 3)
2
+
(y + 3)
2
= 4 in the standard (x, y) coordinate plane?
A. (3,3)
B. (3,3)
C. (
3,
3)
D. (3, 3)
E. (
3,
3)
28. In the standard (x, y) coordinate plane, what is the
length of the line segment that has endpoints (3,4)
and (5,6)?
F. 9
G. 2
41
H. 18
J. 20
2
K. 40
29. A triangle has sides of length 4.7 meters and 9 meters.
Which of the following CANNOT be the length of the
third side, in meters?
A. 5
B. 7
C. 8
D. 11
E. 14
30. If
n
x
n
y
= n
2
for all n $= 0, which of the following must
be true?
F. x + y = 2
G. x y = 2
H. x × y = 2
J. x ÷ y = 2
K.
xy = 2
GO ON TO THE NEXT PAGE.
116 PRACTICE TEST 2 MATHEMATICS TEST
2 2
DO YOUR FIGURING HERE.
31. In the standard (x, y) coordinate plane, what is the
y-intercept of the line given by the equation
3x + 5y = 8?
A. 3
B.
5
3
C.
8
5
D.
3
5
E. 3
32. There are 16 ounces in one pound. If 3.4 pounds of beef
cost $4.95, what is the cost per ounce, to the nearest
cent?
F. $0.09
G. $0.31
H. $1.05
J. $1.46
K. $10.99
33.
1
2
!
2
+
1
3
!
2
+
1
4
!
2
= ?
A.
1
29
B.
3
29
C.
61
144
D.
15
32
E. 9
34. One route along flat terrain from Hermansville to
Melville is to drive straight north from Hermansville
for 120 miles to Jamestown, then, at Jamestown, to
drive straight west for 80 miles to Melville. If a straight,
flat road existed between Hermansville and Melville,
approximately how many miles long would it be?
F. 200
G. 144
H. 100
J. 98
K. 40
35. In order to clean her aquarium, Stephanie must remove
half of the water. The aquarium measures 30 inches
long, 16 inches wide, and 12 inches deep. The aquarium
is currently completely full. What volume of water, in
cubic inches, must Stephanie remove?
A. 1,440
B. 2,880
C. 4,320
D. 5,760
E. 7,200
GO ON TO THE NEXT PAGE.
PRACTICE TEST 2 MATHEMATICS TEST 117
2 2
DO YOUR FIGURING HERE.
36. The bowling league selects its 4 officers by first selecting
the president, then the vice president, then the secretary,
then the treasurer. If there are 40 bowlers who are eli-
gible to hold office and no member can hold more than
one office, which of the following gives the number of
different possible results of the election?
F. 37
4
G. 39
4
H. 40
4
J. 39 × 38 × 37 × 36
K. 40 × 39 × 38 × 37
37. The points R (2,2) and S (6,3) in the standard (x,y) coor-
dinate plane below are 2 vertices of triangle RST, which
has a right angle at S. Which of the following could be
the third vertex, T?
A. (5,7)
B. (5,5)
C. (4,6)
D. (4,9)
E.
4,
9
2
!
38. What value of x will satisfy the equation
0.2(x 2,700) = x?
F. 675
G. 540
H. 0
J. 540
K. 675
39. If 0
x 90
and tan x =
15
8
, then cos x = ?
A.
8
17
B.
15
17
C.
17
8
D.
17
15
E.
8
15
GO ON TO THE NEXT PAGE.
118 PRACTICE TEST 2 MATHEMATICS TEST
2 2
DO YOUR FIGURING HERE.
40. A square pool with an area of 81 square feet is to be
placed entirely within a circular enclosure with a radius
of 10 feet. Tiles will be laid within the entire enclosure
around the pool (but not under it). What is the approx-
imate area, in square feet, of the enclosure that will be
tiled?
F. 81
G. 233
H. 315
J. 396
K. Cannot be determined without knowing the exact
placement of the pool.
41. In the standard (x,y) coordinate plane, which of the
following lines goes through (3,4) and is parallel to
y = 2x + 2?
A. y =
1
2
x + 2
B. y = 2x 2
C. y = 2x + 4
D. y = 2x + 10
E. y = 3x + 2
42. In the figure below, tan ϕ = ?
F.
1
2
G.
2
H. 1
J. 3
K. 3
2
43. Which of the following operations will produce the
smallest result when substituted for the blank in the
expression:
2
3
____ 3?
A. plus
B. minus
C. multiplied by
D. divided by
E. averaged with
GO ON TO THE NEXT PAGE.
PRACTICE TEST 2 MATHEMATICS TEST 119
2 2
DO YOUR FIGURING HERE.
44. The value of b that will make
b
3
+2 =
1
4
a true statement
lies between which of the following numbers?
F. 4 and 6
G. 1 and 3
H. 1 and 1
J. 1 and 3
K. 3 and 5
45. What is the solution set of |3a 2| 7?
A. {a: a 3}
B.
"
a:
5
3
a 3
#
C.
"
a:
5
3
a 3
#
D.
"
a:
5
3
a 3
#
E.
"
a:
5
3
a 3
#
46. When measured from a point on the ground that is a
certain distance from the base of a cell phone tower,
the angle of elevation to the top of the tower is 41
,
as shown below. The height of the cell phone tower is
200 feet. What is the distance, in feet, to the cell phone
tower?
F. 200 tan 41
G. 200 sin 41
H. 200 cos 41
J. 200 sec 41
K. 200 cot 41
47. For the area of a square to triple, the new side lengths
must be the length of the old sides multiplied by:
A.
3
B. 3
C. 4
D. 2
3
E. 9
GO ON TO THE NEXT PAGE.
120 PRACTICE TEST 2 MATHEMATICS TEST
2 2
DO YOUR FIGURING HERE.
48. The volume of a cube is given by the formula s
3
, where
s is the length of a side. If a cube has a volume of 64,
and the length of each side is halved, the new cube’s
volume will be:
F. 3
G. 6
H. 8
J. 16
K. 32
49. In the parallelogram below, lengths are given in inches.
What is the area of the parallelogram, in square inches?
A.
94
B. 7
47
C. 49
D. 63
E. 16
47
50. If 8a
6
b
3
< 0, then which of the following CANNOT be
true?
F. b < 0
G. b > 0
H. a = b
J. a < 0
K. a > 0
51. If log
4
x = 3, then x = ?
A.
1
log
12
B. 4 log
3
C. 12
D. 64
E. 81
GO ON TO THE NEXT PAGE.
PRACTICE TEST 2 MATHEMATICS TEST 121
2 2
DO YOUR FIGURING HERE.
52. If a system of 2 linear equations in 2 variables has NO
solution, and 1 of the equations is graphed in the (x,y)
coordinate plane below, which of the following could
be the equation of the other line?
F. y = 2
G. y =
1
4
x + 2
H. y = 2x 4
J. y =
4
3
x + 2
K. y = 4x 4
53. In a game, 80 marbles numbered 00 through 79 are
placed in a box. A player draws 1 marble at random from
the box. Without replacing the first marble, the player
draws a second marble at random. If both marbles drawn
have the same ones digit (that is, both marbles have a
number ending in 0, 1, 2, 3, etc.), the player is a winner.
If the first marble drawn is numbered 35, what is the
probability that the player will be a winner on the next
draw?
A.
1
79
B.
7
80
C.
7
79
D.
1
10
E.
8
79
GO ON TO THE NEXT PAGE.
122 PRACTICE TEST 2 MATHEMATICS TEST
2 2
DO YOUR FIGURING HERE.
54. In the standard (x,y) coordinate plane, what is the equa-
tion of the line that passes through the origin and the
point (3,4)?
F. y =
1
4
x +
3
4
G. y =
1
4
x
1
3
H. y =
4
3
x
J. y =
1
2
x +
3
4
K. y =
9
4
x
55. The measure of the vertex angle of an isosceles triangle
is (a + 30)
. The base angles each measure (2a 15)
.
What is the measure in degrees of one of the base angles?
A. 36
B. 45
C. 57
D. 66
E. 90
56. What is the smallest possible value for the product of
2 integers that differ by 7?
F. 8
G. 0
H. 6
J. 10
K. 12
57. Three distinct lines, all contained within a plane, sepa-
rate that plane into distinct regions. What are all of the
possible numbers of distinct regions of the plane that
could be separated by any such three lines?
A. 4, 6, 7
B. 4, 5, 6
C. 3, 5, 7
D. 3, 5, 6
E. 3, 4, 5
GO ON TO THE NEXT PAGE.
PRACTICE TEST 2 MATHEMATICS TEST 123
2 2
DO YOUR FIGURING HERE.
58. Given the vertices of parallelogram QRST in the stan-
dard (x, y) coordinate plane below, what is the area of
triangle QRS, in square units?
F. 24
G. 28
H. 48
J. 60
K. 80
59. The first and second terms of a geometric sequence are
a and ab, in that order. What is the 643rd term of the
sequence?
A. (ab)
642
B. (ab)
643
C. a
642
b
D. a
643
b
E. ab
642
60. Points A, B, and C are three distinct points that lie on
the same line. If the length of AB is 19 meters and the
length of BC is 13 meters, then what are all the possible
lengths, in meters, for AC?
F. 6 only
G. 32 only
H. 6 and 32 only
J. Any number less than 32 or greater that 6
K. Any number greater than 32 or less than 6
END OF THE MATHEMATICS TEST.
STOP! IF YOU HAVE TIME LEFT OVER, CHECK YOUR WORK ON THIS SECTION ONLY.
124 PRACTICE TEST 2 READING TEST
3 3
READING TEST
35 Minutes—40 Questions
DIRECTIONS: This test includes four passages, each
followed by ten questions. Read the passages and choose
the best answer to each question. After you have selected
your answer, fill in the corresponding bubble on your
answer sheet. You should refer to the passages as often
as necessary when answering the questions.
PASSAGE I
PROSE FICTION: The Summer Sandwich Club
Maxwell was one of those kids I know I will
remember for the rest of my life. I first met Max three
summers ago when he showed up at the park on the first
day of camp with his mother Katherine. After a brief
5 good morning, he went off to play with the rest of the
five and six year olds who I would be counseling for the
next several weeks. As his mother walked back to her
car, I couldn’t help but notice that she looked as though
she had just finished running a marathon; however, that
10 thought left my mind soon after she drove away, as I
was surrounded by the smiling faces of thirty brand
new campers.
The summer started off great, and Max and I hit it
off right away; he looked up to me as an older brother,
15 and I thought he was a great little kid. There were a
few things that stuck out in my mind as odd, though,
like when he would complain of being hungry an hour
or so after lunch at least once or twice a week. By the
third week of camp, I decided that it was something
20 I needed to investigate, and during lunch time I went
over to his table and asked if I could sit next to him.
He giggled and said, “Sure Jake,” feeling special that
I would want to spend my lunch break with him. His
lunch consisted of a bag of potato chips, a can of soda,
25 and a chocolate bar—hardly a healthy meal for a five
year old. I offered him half of my sandwich and his
eyes lit up like it was his birthday.
That afternoon, when Katherine came to pick Max
up from the park, I pulled her aside to discuss the lunch
30 issue.
“Katherine, Maxwell needs to have a healthy
lunch.” She looked down at the ground.
“What do you mean, Jake?”
“I mean Max can not keep eating junk food
35 every day.”
“Oh. That. I’m sorry about that. It’s just that I
work back-to-back jobs every night and barely make it
home in time to get him out of bed and dressed before
camp starts in the morning. His babysitter is supposed
to40 pack Max’s lunch for him at night when she puts
him to bed. We have had a couple of new babysitters
lately, and sometimes they forget to do it, so I end up
having to throw something together at the last minute.
I’ll make sure it doesn’t happen any more.”
45 “It happens to the best of us; I just wanted to
make sure you knew what he was eating. After a couple
days of him being hungry I got worried and wondered
who was making his lunch for him. See you tomorrow
morning then.”
50 Several days later I expected to see Max eating
a sandwich went I went over to him at lunchtime. His
lunch once again consisted entirely of junk food. Some-
thing had to change; at the very least he needed to be
eating much less sugar.
55 “Katherine,” I called to his mother as she stepped
out of her car that afternoon. I really had no idea what
I was supposed to say. It was quite a predicament. “We
really need to fix this problem with Max’s lunch.”
“Jake, I know, it’s just that the house payment was
60 due yesterday, and I haven’t had the, uh, time to get
to the grocery store,” she trailed off. “Things are just
a little hard for us right now.”
She was obviously self-conscious at the moment,
and I felt bad for having brought it up again. I told
65 her that I had a plan, and not to worry about it. After
explaining what I meant, the look on her face was one
of relief and thanks, and she and Max headed home for
the day.
For the rest of the summer, I spent my lunches
70 with Max and his friends, having meetings of what we
called the “Sandwich Club”: every day I would bring a
couple of extra sandwiches, and anybody who wanted
to try one could have some. Max never seemed to care
what kind of sandwiches I brought to the club, but just
75 giggled and smiled up at me every afternoon.
At the end of the summer, I got a letter from
Katherine, thanking me for being so kind to she and
Max. I wrote back telling her that I could hardly wait
until the next meeting of the “Sandwich Club,” and to
80 tell Max that I said hello.
For the next two years, the “Sandwich Club” had
regular meetings, Monday-Friday at noon, all summer
long. After that, Max and his mother moved to be closer
to his grandparents, and I went back to having my lunch
GO ON TO THE NEXT PAGE.
PRACTICE TEST 2 READING TEST 125
3 3
85 with the rest of the staff. But for those few years, the
“Summer Sandwich Club” brought joy to one camp
counselor and many young campers.
1. When Jake says, “It happens to the best of us,” he is
primarily saying that:
A. he understands that sometimes things happen that
are beyond our control.
B. Katherine is a perfect parent, and he is surprised that
Max is unhappy.
C. Max is a picky eater and would not eat a healthy
lunch anyway.
D. Max is his favorite camper, despite the problems
faced by Max’s mother.
2. It can be reasonably inferred from the conversations
with Jake, that Katherine:
F. is a stay-at-home-mother.
G. does not care about her son.
H. works two jobs to make ends meet.
J. believes that junk food is healthy.
3. The idea that Jake’s mother is trying her best to take care
of her son is least supported by which of the following
quotations from the passage?
A. “It’s just that I work back-to-back jobs every night
and barely make it home in time to get him out of
bed and dressed before camp starts in the morning.”
B. “Katherine, Maxwell needs to have a healthy
lunch.”
C. “Things are just a little hard for us right now.”
D. “I’ll make sure it doesn’t happen any more.”
4. As it is used in line 57 the word predicament most nearly
means:
F. joke.
G. solution.
H. complaint.
J. challenge.
5. It can be inferred from the passage that Jake is:
A. Max’s older brother.
B. dissatisfied with his job.
C. a good influence on Max.
D. someone Max barely knows.
6. The passage makes it clear that the “Sandwich Club”:
F. lasted as long as Max was a camper.
G. met only when it rained.
H. was an insult to Katherine.
J. was Max’s favorite part of camp.
7. You may reasonably infer from the details in the passage
that Katherine and Max:
A. dislike Jake.
B. are very wealthy.
C. do not trust other people.
D. have little money.
8. Katherine can most accurately be characterized as:
F. indifferent and withdrawn.
G. caring but distracted.
H. cruel and arrogant.
J. friendly but aloof.
9. The word issue
, as it is used in line 30, most nearly
means:
A.
publication.
B. incident.
C. idea.
D. problem.
10. The
title,
“The Summer Sandwich Club,” combined
with details presented in the passage imply that:
F. everyone loves sandwiches.
G. Jake only eats sandwiches in the summer.
H. children should join clubs to make friends.
J. the club was created because of Max.
GO ON TO THE NEXT PAGE.
126 PRACTICE TEST 2 READING TEST
3 3
PASSAGE II
SOCIAL SCIENCE: Lewis and Clark Go West
Over two hundred years ago, at the request of
President Jefferson, the corps of volunteers for “North
Western Discovery” set off under the command of
Meriwether Lewis and William Clark to find the fastest
5 water route across North America. The path they were
to carve out would be the first of its kind; they were
setting a course through the territory of potentially dan-
gerous Indian tribes and ferocious animals. None but
the fearless and inventive, the most resourceful and
10 curious, would dare to undertake such a venture. In
1803, virtually no one had attempted to cross the stretch
of land between the mighty Mississippi and the vast
Pacific Ocean using only water routes. All of the won-
ders of those states in the West are, in part, the result
15 of this expedition. These intrepid pioneers, especially
Lewis and Clark, deserve to be remembered now some
two centuries after their courageous journey into the
unknown lands west of the Mississippi. The rolling
hills of the breadbasket, the ski-resorts in the snow-
20 capped Rocky Mountains, and the lush, fertile valleys
of the coast echo the bravery of all those involved.
After receiving wilderness training in Washington
D.C., Meriwether Lewis set out on July 5, 1803, picked
up guns at Harpers Ferry, Virginia, and then moved to
Pittsburgh25 to pick up a 55-foot keelboat. Floating it
down the Ohio, he met with Clark in Indiana, who
took over command of the boat and crew, while Lewis
then rode on to get supplies in St. Louis. Months later,
in May, the entire party gathered in St. Louis. The
30 forty-some men were to travel from there to the Pacific
Ocean in only the keelboat and two smaller boats, all of
which were moved by sails, towropes, poles, or oars.
The beginning of their journey was a voyage
of confirmation; traders had gathered information of
35 various possible water routes to the Pacific, and Lewis
and Clark’s job was to confirm the truth of such reports
and observe anything else of importance along the way.
They also catalogued new species of plants and animals
which they encountered, and worked toward peace with
40 several Indian tribes. History tells us that the few mes-
sages the men were able to send back told of their
health and high spirits. They were all eager to explore
just what might lie beyond the Mississippi.
Despite having adequate supplies and equipment,
45 including guns, the men’s journey was still a danger-
ous one. They were traversing the wild and until this
point, the only other individuals to have crossed it were
fur traders and trappers. It was largely Indian terri-
tory and although most tribes, such as the Otos, the
50 Missouris, and the Mandans were friendly, the Sioux
and the Blackfeet tried to impede the group’s progress
on more than one occasion. Illness claimed the life of
one man early, but despite the strenuous pace of the
expedition, there were no further losses.
55 Throughout it all, including long winters and the
harsh conditions of wildness living, the travelers con-
tinued to forge west in search of an efficient trade route
using only the rivers. In September of 1806, some three
years after they started on their voyage, Lewis, Clark,
60 and their team made it to the Pacific Ocean. Relying on
the Missouri and Columbia rivers as their main “high-
ways,” and taking the help of friendly Indian tribes
whenever they could, the expedition was a success,
and served as an example for all manner of westward
65 expansion.
Despite the success of their expedition, proving
that there was indeed a water route from the Missis-
sippi River to the Pacific Ocean, future travelers to the
West found faster passage on land, utilizing the Oregon
70 Trail. Keelboats were eventually replaced by covered
wagons and trains, and America pushed ever onward
into the West. The settlers who came after Lewis and
Clark went forward with blind-devotion knowing then
that it could be done. The initial breakthrough into that
75 unknown land was all that the country really needed.
From there on out, the rest was history.
11. One of the main points that the author seeks to make in
the passage is that westward expansion:
A. was never attempted prior to the Lewis and Clark
expedition.
B. was a challenging but important aspect of the growth
of the United States.
C. led to the discovery of many new and dangerous
Indian tribes.
D. resulted in the development of the corps of volun-
teers for “North Western Discovery.”
12. The focus of the passage can best be summarized as a
study of both the:
F. Lewis and Clark Expedition and the characteristics
of the United States in the early 1800s.
G. history of Midwest development and the Lewis and
Clark Expedition.
H. Lewis and Clark Expedition and the legendary
Northwest Passage.
J. losses and difficulties faced by the Lewis and Clark
Expedition.
13. According to the information presented in the passage,
which of the following best describes the relationship
between the Lewis and Clark Expedition and the settlers
who came after them?
A. Everyone
to follow the Expedition used Lewis and
Clark’s water route.
B. The settlers who went west after the Expedition were
much more cautious.
C. Both the Lewis and Clark expedition and the future
settlers suffered great losses.
D. The Lewis and Clark Expedition gave others confi-
dence to head West.
14. According to the passage, the motivation for the Lewis
and Clark Expedition was to:
F. make money.
G. catalog the animals of North America.
H. discover a water route to the Pacific.
J. reach the Rocky Mountains.
GO ON TO THE NEXT PAGE.
PRACTICE TEST 2 READING TEST 127
3 3
15. As it is used in the 2nd paragraph (lines 22–32), the
word party most nearly means:
A. a joyous celebration.
B. a group of people setting out on a trip.
C. a segment of the population.
D. a meeting to discuss business matters.
16. As it is depicted in the passage, the initial mood of the
Lewis and Clark expedition can best be described as:
F. hopelessly discouraged.
G. eagerly determined.
H. remarkably cautious.
J. overtly happy.
17. It can be inferred that the word forge as it is used in
Paragraph 5 (lines 55–65) refers to:
A. creating new tools out of metal.
B. searching for food.
C. continuing a journey.
D. crossing a river on foot.
18. According to the passage, which of the following were
the primary dangers faced by the Lewis and Clark
expedition?
F. Illness and lack of motivation.
G. Fast moving water.
H. Wagons that fell apart.
J. Conflicts with the indigenous people.
19. As it relates to the passage, all of the following were
methods used to move the boats EXCEPT:
A. man power.
B. wind power.
C. rowing power.
D. steam power.
20. According to the passage, in the early part of their jour-
ney members of the Lewis and Clark Expedition were
doing all of the following EXCEPT:
F. receiving wilderness training.
G. cataloguing new species of plants and animals.
H. confirming possible water routes across the conti-
nent.
J. sending back messages regarding their status.
GO ON TO THE NEXT PAGE.
128 PRACTICE TEST 2 READING TEST
3 3
PASSAGE III
HUMANITIES: Colorful Reflections on Fairfield Porter
My first encounter with the international artist
and art critic Fairfield Porter was actually through the
poetry of his wife, Anne (Channing) Porter. While
both grew to become quite celebrated in their crafts,
5 Fairfield’s story is unique.
Born into an affluent, artistic family in 1907, the
boy who was to one day become a renowned artist
and respected art critic showed a comparative lack of
artistic ability when seen next to his siblings. While
10 his older brother Eliot took to photography, Fairfield
Porter, despite being remarkably intelligent, appeared
to be lacking any natural artistic talents. It seemed
that, although a member of a family full of artists, his
true skill lay in the critiquing of others’ artistry. This
15 was evidenced in his second year at Harvard by Fair-
field’s decision to pursue art history as his major field of
study. After studying at Harvard under Arthur Pope and
then traveling briefly through Europe, Fairfield came
back to the United States to further his education at
20 the Art Students League in New York City. There he
became acquainted with the famed photographer Alfred
Stieglitz—the work of whom is said to have positively
influenced Fairfield’s paintings to some degree.
Between the years 1931 and 1932, Fairfield spent
25 the majority of his time in Italy learning to appreci-
ate and critique the works of the great Renaissance
painters. His training came from both direct study
under world-famous art historian Bernard Berenson,
and from countless hours spent in museums and
30 galleries observing the greatest pieces of Italian art.
Following his marriage to Anne upon his return
from Italy, Fairfield spent the better part of the next two
decades developing his skills as a painter while caring
for his autistic son. During this period his meetings with
35 the French Intimist painter Willem De Kooning would
prove to have a profound effect on his later works.
Porter was the first to publicly acclaim the work of
Kooning.
In fact, what made Porter so famous was his knack
40 for responding directly to an artist’s work. He found
fault with the common “talk based” criticism that spoke
to art only in reference to its past or to some vague the-
oretical framework; such criticism attempted to shape
the future of art and was far too biased for Porter.
45 His time as an art critic for such publications as Art
News and The Nation ended, however, in 1961 when
he decided to pursue a full-time painting career.
The other side of his fame, his uncommon
approach to painting, is just as important to the
50 understanding of Fairfield Porters contributions to the
world of art. His personal philosophy comes from a
blending of two views; art should be personal, emo-
tional, and representative of its subject, while at the
same time be boldly colorful, expressive, and gener-
ally55 abstract. Drawing on his vast knowledge of art
history, especially the styles of French Intimism, Porter
fused these two feelings to create a powerful, emotive
collection of paintings about families, individuals, and
the home, as well as moving nature scenes such as
60 The Door to the Woods (1971) and Maine Toward the
Harbor (1967).
When he died in 1975, on a morning walk along
the ocean, he left the world as one of the most respected
art critics in the past century. On top of that, his work
65 as a painter is still viewed within the art community
as amazingly distinctive and especially representative
of his life. It is sad to say that now, however, some
thirty years after his death, he is still virtually unknown
outside of art circles. This remarkably insightful, artic-
ulate,70 creative individual needs to be discovered by the
common man and revered for his continuing influence
on the artists of today. The words of this intellectual
were some of the best and most honest critiques of art
ever spoken.
21. The main purpose of the passage can best be described
as an attempt to:
A. explain Porters renowned ability to candidly
address artists’ works.
B. illustrate the influence several renowned artists had
on the works of Porter.
C. appraise Porters unusual methods of painting and
critiquing artwork.
D. chronicle Porters life, particularly the events and
beliefs that shaped his career.
22. The authors attitude towards the subject of the passage
can best be characterized as:
F. detached interest.
G. amused tolerance.
H. warm appreciation.
J. deep abhorrence.
23.
As described in the passage, Porters method of crit-
icizing
art can best be summarized by which of the
following statements?
A. Porters criticisms were frank and forthright, and
were based solely on his evaluation of the piece of
art that he was appraising.
B. Porter criticized art based on the context of the paint-
ing and conceptual structures that he found most
useful in his evaluations.
C. Porters critiques were comparable to those of
Bernard Berenson, who greatly influenced Porters
outlook on art.
D. Porter targeted his criticisms at helping artists by
attempting to influence their forthcoming works.
24. Porters painting style can be described by all of the
following EXCEPT:
F. stirring.
G. vivid.
H. trite.
J. individualistic.
GO ON TO THE NEXT PAGE.
PRACTICE TEST 2 READING TEST 129
3 3
25. Without the first paragraph, the passage would lose:
A. an overview of the passage as a whole.
B. a brief introduction and transition into the topic.
C. important detail that later becomes relevant to the
passage.
D. an explanation of the logic behind the authors
viewpoint.
26. In line 11, the statement “despite being remarkably
intelligent” is intended to:
F. call attention to the fact that although Porter was
a well-respected art critic, he failed to impress his
college professors.
G. communicate to the reader that Porter’s lack of a
formal education did not detract from his ability to
critique art.
H. emphasize to the reader that Porters high level of
intelligence was not related to his artistic ability.
J. inform the reader that Porters position as an art
critic was so difficult that it challenged his intellect.
27. The word revered in line 71 most nearly means:
A. trusted.
B. depreciated.
C. reminiscent.
D. honored.
28. According to the passage, when did Fairfield Porter
become serious about becoming an artist?
F. Immediately upon his return from Italy.
G. While he was studying at the Art Students League.
H. Just before his death in 1975.
J. Approximately thirty years after he returned from
Italy.
29. The third paragraph states that, during 1931 and 1932,
Fairfield Porter was:
A. continuing his training as an art critic.
B. the greatest art critic in Italy.
C. planning his marriage to Anne Channing.
D. training to become a Renaissance painter.
30. The author uses the phrase “other side of his fame”
(line 48) most likely in order to:
F. suggest that Fairfield Porter was better known as an
artist than as an art critic.
G. indicate that Fairfield Porter was both a renowned
art critic and painter.
H. show that Fairfield Porter was not aware of his
popularity as a painter.
J. suggest that other art critics of the time were more
famous than was Fairfield Porter.
GO ON TO THE NEXT PAGE.
130 PRACTICE TEST 2 READING TEST
3 3
PASSAGE IV
NATURAL SCIENCE: This passage discusses some of
the controversy surrounding the existence of dark
matter in the universe.
Dark matter in the universe is believed by some
scientists to be a substance that is not readily observable
because it does not directly refract light or energy. Its
existence can only be deduced because of the effect that
5 it has on surrounding matter. In fact, some members of
the scientific community have argued that dark mat-
ter does not actually exist. Others, however, believe in
its existence, in part because the scientific community
does not have a complete understanding of gravita-
tional10 science. On the other hand, some would argue
that it is the understanding of gravitational science that
leads most scientists to believe in the existence of dark
matter, because without dark matter, there are many
cosmological phenomena that are difficult to explain.
15 For example, dark matter in the universe may
have a peculiar effect on the Milky Way galaxy. Some
scientists believe that the interaction between dark mat-
ter and other smaller, nearby galaxies is causing the
Milky Way galaxy to take on a warped profile. It has
20 been asserted that not only does dark matter exist, it
may also be responsible for the Milky Way’s unusual
shape. The interaction referenced involves two smaller
galaxies near the Milky Way, called Magellanic clouds,
moving through an enormous amount of dark matter,
25 which, in effect, enhances the gravitational pull that the
two Magellanic clouds could have on the Milky Way
and other surrounding bodies. Without the existence of
the dark matter, the Magellanic clouds would not have
sufficient mass to have such a strong effect on the bend
30 of the Milky Way galaxy.
The strongest evidence for the validity of this
hypothesis rests in Newtonian physics, and the hypoth-
esis that anything with mass will exert a gravitational
pull. The Milky Way and other galaxies with pecu-
liar35 warped shapes are being molded by a gravitational
force. However, there is nothing readily observable
with sufficient mass that could cause such a high level
of distortion via gravitational pull in the vicinity of
the Milky Way. Therefore, something that is not easily
40 observed must be exerting the necessary force to create
the warped shape of the galaxy.
Aaron Romanowsky and several colleagues have
questioned the effect that dark matter might have on
galaxies. They point to the existence of several ellipti-
cal45 galaxies surrounded by very little dark matter as
evidence that dark matter is not, in fact, the cause
of the warped galaxies. While they do not claim that
their findings should be interpreted to conclude that
dark matter does not exist, they apparently believe
50 that the results of their studies cast doubt on some
of the conventional theories of galaxy formation and
manipulation.
Several models constructed by researchers from
the University of California at Berkeley, however,
55 point to the idea that dark matter is the most likely
explanation for the distorted shape of the Milky Way
and other galaxies. Using computer models, they have
mapped the likely interactions between certain galax-
ies and the surrounding dark matter, and those models
60 have shown not only the possibility that dark matter
is responsible for the warped shape of the Milky Way,
but that the relationship between the dark matter and
the Magellanic clouds is dynamic; the movement of
the clouds through the dark matter seems to create a
65 wake that enhances their gravitational influence on the
Milky Way.
31. As it is used in line 14, the term phenomena most nearly
means:
A. occurrences.
B. problems.
C. attitudes.
D. surprises.
32. The passage states that some members of the scientific
community are reluctant to believe in the existence of
dark matter because:
F. there is absolutely no evidence for the existence of
dark matter.
G. no one understands how to apply gravitational
science.
H. dark matter cannot be directly observed.
J. dark matter has little effect on surrounding matter.
33. What does the passage offer as evidence for the exis-
tence of dark matter?
A. A complete understanding of gravitational science.
B. The enormous mass of Magellanic clouds.
C. The shape of the Milky Way galaxy.
D. A photograph taken with the aid of a refracting
telescope.
34.
According to the passage, what is Aaron Romanowsky’s
theory
regarding
dark matter?
F. It cannot be conclusively proven that dark matter
affects the shape and formation of galaxies.
G. The discovery of certain galaxies disproves the
theory that dark matter exists in the universe.
H. Computer models suggest that dark matter is respon-
sible for warped galaxies.
J. Dark matter has not effect at all on the shape of a
galaxy.
GO ON TO THE NEXT PAGE.
PRACTICE TEST 2 READING TEST 131
3 3
35. The last paragraph supports the general hypothesis
provided earlier in the passage that:
A. the effect of Magellanic clouds on galaxies is
enhanced by dark matter.
B. computer models are necessary for an understanding
of gravitational science.
C. dark matter has little to no effect on the formation
of certain cosmological phenomena.
D. the shape of the Milky Way galaxy can be deduced
by observing the matter surrounding it.
36. The main purpose of the third paragraph is to point out
that:
F. dark matter was first discovered by applying
Newtonian physics.
G. different viewpoints exist regarding gravitational
science.
H. galaxies with peculiar shapes could not exist in the
presence of dark matter.
J. scientific theories provide support for the existence
of dark matter in the universe.
37. The word conventional in line 51 most nearly means:
A. easily understood.
B. formally disputed.
C. strictly interpreted.
D. generally accepted.
38. Which one of the following is NOT mentioned in the
passage as a scientific theory regarding dark matter?
F. The existence of dark matter cannot be proved by
direct observation.
G. Dark matter may be responsible for the shape of the
Milky Way.
H. It is certain that dark matter has no influence on
surrounding celestial bodies.
J. Magellanic clouds require the presence of dark
matter in order to influence the shape of galaxies.
END OF THE READING TEST.
STOP! IF YOU HAVE TIME LEFT OVER, CHECK YOUR WORK ON THIS SECTION ONLY.
39. According to the passage, dark matter cannot be readily
detected because:
A. dark matter does not actually exist.
B. most of the dark matter in the universe is hidden
behind galaxies.
C. it does not directly interact with light or energy.
D. it has no effect on the surrounding matter.
40. The passage supports which of the following statements
about dark matter?
F. Its existence is inferred by some researchers based
on observations of cosmological bodies composed
of ordinary matter.
G. Its existence has been conclusively proven by com-
puter models.
H. If it does not exist, the universe is largely empty.
J. Its presence is readily observable to researchers who
completely understand how to apply gravitational
science.
132 PRACTICE TEST 2 SCIENCE REASONING TEST
4
) ) ) ) ) ) ) ) )
4
SCIENCE REASONING TEST
35 Minutes—40 Questions
DIRECTIONS: This test includes seven passages, each
followed by several questions. Read the passage and
choose the best answer to each question. After you have
selected your answer, fill in the corresponding bubble
on your answer sheet. You should refer to the passages
as often as necessary when answering the questions.
You may NOT use a calculator on this test.
PASSAGE I
A researcher has conducted two experiments to test the
rate of pinecone production in the Pinus palustris Miller
(a type of pine tree).
Experiment 1
P. palustris Miller seeds were collected from 5 different
populations (A1, A2, A3, A4, A5) each of which was from
a different site (S1, S2, S3, S4, S5).
The seeds were grown under controlled conditions in
a greenhouse. 300 of these seedlings from each popula-
tion were chosen at random. Each set of seedlings was
divided into 30 groups with 10 seedlings in each group.
The seedlings were planted in marked cylindrical contain-
ers which were then placed at each of the 5 sites. Figure 1
shows the procedure for A1.
Figure 1 25 Cups containing a total of 250 A1 seedlings
Table 1 shows the number of pinecones that were pro-
duced on each tree.
The researchers also collected data on the root structure
of the trees. From the information they collected they came
up with the following formula relating the root structure in
inches to the number of pinecones produced:
number of pinecones = 0.037 + 0.147 (root thickness)
Statistical analysis indicated that this equation was accurate.
Table 1
Pinecones produced per tree
Site A1 A2 A3 A4 A5
S1 2.1 7.1 12.0 2.4 3.1
S2 3.9 2.5 8.5 6.2 6.4
S3 0.4 6.7 3.1 9.3 7.2
S4 5.2 2.1 2.9 0.2 4.5
S5 1.8 6.3 0.9 3.7 8.5
Experiment 2
P. palustris Miller seeds were collected and grown in
the same manner as in Experiment 1. When the seeds had
grown into seedlings, 150 containers were prepared with 5
A1 seedlings and 5 seedlings from either A2, A3, A4 or
A5. Seven containers for each of the 4 combinations were
planted at each site.
Table 2 shows how many pinecones were produced on
each A1 plant.
Table 2
Pinecones produced per A1 tree when
planted with
Site A2 A3 A4 A5
S1 5.7 3.2 6.7 3.5
S2 3.2 1.7 4.3 5.2
S3 9.6 8.4 0.8 7.0
S4 4.2 3.2 1.3 0.2
S5 4.9 6.1 6.1 3.9
GO ON TO THE NEXT PAGE.
PRACTICE TEST 2 SCIENCE REASONING TEST 133
4
) ) ) ) ) ) ) ) )
4
1. In Experiment 1, trees from A5 produced more
pinecones than did trees from A4 at which of the
following sites?
A. S4 only
B. S1 and S5 only
C. S1, S2, S4, and S5 only
D. S1, S2, S3, S4, and S5 only
2. In Experiment 1, A1 trees produced the largest number
of pinecones at which of the following sites?
F. S1
G. S3
H. S4
J. S5
3. The procedures utilized in Experiment 2 were repeated,
except that only 25 containers were planted at a sixth
site (S6). The results appear in Table 3.
Table 3
Pinecones produced per A1 tree
when planted with
Site A2 A3 A4 A5
S6 4.1 6.4 1.9 0.3
Based on these data, one should conclude that A1 trees
produced more pinecones at S6 than at which of the
following sites in Experiment 2?
A. S1
B. S3
C. S4
D. S5
4. A student wanted to produce the greatest number of
pinecones from 6 A1 trees, using the procedures from
Experiment 2. Which plants and site should the A1 trees
be combined with to achieve the desired results?
F. A4 and S1
G. A2 and S3
H. A3 and S2
J. A5 and S5
5. In which of the following ways was Experiment 2
different from Experiment 1?
A. Experiment 2 included trees from more than 1 pop-
ulation.
B. Experiment 2 combined trees from more than
1 species.
C. Experiment 2 trees were planted at all 5 sites.
D. Experiment 2 trees were planted at only 1 site.
6. In Experiment 2, how many seedlings were planted in
each container?
F. 6
G. 8
H. 10
J. 12
GO ON TO THE NEXT PAGE.
134 PRACTICE TEST 2 SCIENCE REASONING TEST
4
) ) ) ) ) ) ) ) )
4
PASSAGE II
Researchers conducted trials on a certain prescription
drug delivered in immediate-release capsules and extended-
release capsules.
Figure 1 shows the mean concentration (nanograms per
milliliter [ng/mL]) of the two active ingredients of the
prescription drug in patients’ blood plasma over time (hr).
Figure 1
Table 1
Body system Side effect Prescription drug group (%) Placebo group (%)
General
Feeling of weakness 6 5
Headache 26 14
Digestive system
Loss of appetite 32 5
Diarrhea 8 0
Dry mouth 31 5
Nausea 14 0
Nervous system
Anxiety 7 4
Dizziness 9 0
Insomnia 25 11
Irritability 11 4
Cardiovascular system Rapid heart rate 10 2
Nutritional Weight gain 15 0
In clinical trials of the prescription drug, subjects given
the prescription drug were interviewed at regular intervals
about the symptoms the prescription drug is meant to relieve.
After each interview, the subjects were assigned a symptom
score. A high symptom score corresponds to high intensity of
symptoms, and a low symptom score indicates low intensity
of symptoms. Figure 2 shows the mean symptom score over
time (hr) for subjects who took the prescription drug.
In the clinical trials, some subjects were given the pre-
scription drug and some subjects were given a placebo (an
inactive pill). Table 1 shows the percentage of subjects from
both groups who reported various adverse side effects.
Figure 2
GO ON TO THE NEXT PAGE.
PRACTICE TEST 2 SCIENCE REASONING TEST 135
4
) ) ) ) ) ) ) ) )
4
7. According to Figure 1, 16 hours after taking the
extended-release form of the prescription drug, the dif-
ference in mean blood plasma concentration between
Ingredient A and Ingredient B is closest to:
A. 7 ng/ml.
B. 9 ng/ml.
C. 11 ng/ml.
D. 16 ng/ml.
8. Based on the data in Figures 1 and 2, the researchers
should make which of the following conclusions about
the overall change in mean blood plasma concentration
and mean symptom score over time following dosage?
F. Both mean blood plasma concentration and mean
symptom score increase then decrease.
G. Both mean blood plasma concentration and mean
symptom score decrease then increase.
H. Mean blood plasma concentration increases then
decreases, and mean symptom score decreases then
increases.
J. Mean blood plasma concentration decreases then
increases, and mean symptom score increases then
decreases.
9. According to Figure 1, mean blood plasma concentra-
tion of Ingredient A administered in immediate-release
form increases most during which of the following time
periods?
A. From the moment of dosage to 3 hours after dosage.
B. From 3 hours after dosage to 10 hours after dosage.
C. From 10 hours after dosage to 14 hours after dosage.
D. From 14 hours after dosage to 24 hours after dosage.
10. Which of the following conclusions about adverse side
effects caused by the prescription drug is consistent with
the results shown in Table 1?
F. Results from the placebo group most question the
number of instances of feeling of weakness caused
by the prescription drug.
G. Results from the placebo group most question the
number of instances of insomnia caused by the
prescription drug.
H. Results from the placebo group least question the
number of instances of anxiety caused by the pre-
scription drug.
J. Results from the placebo group least question the
number of instances of irritability caused by the
prescription drug.
11. The symptom score of a clinical trial subject given the
extended-release form of the prescription drug remained
unchanged for 8 hours. Based on Figure 2, the 8-hour
period most likely began:
A. 3 hours after dosage.
B. 5 hours after dosage.
C. 9 hours after dosage.
D. 14 hours after dosage.
GO ON TO THE NEXT PAGE.
136 PRACTICE TEST 2 SCIENCE REASONING TEST
4
) ) ) ) ) ) ) ) )
4
PASSAGE III
The atmosphere is made up of 4 distinct layers: the
troposphere, stratosphere, mesosphere, and thermosphere.
Different types of clouds form in the different layers
depending on the pressure in the atmosphere and the ambi-
ent temperature. The cloud types include nimbus, stratus,
cumulus, and cirrus. Figure 1 shows the location of the
Figure 1
12. According to Figure 1, the atmospheric layer with the
greatest range in pressure is the:
F. mesosphere.
G. thermosphere.
H. stratosphere.
J. troposphere.
13. Which of the following statements about the formation
of cumulus clouds is supported by the data presented in
Figure 1? Cumulus clouds typically form in:
A. pressures between 8 and 12 psi and at an average
temperature of 35
C.
B. pressures between 12 and 16 psi and at an average
temperature of 22
C.
C. a pressure of 4 psi and at average temperatures
between 12
C and 22
C.
D. a pressure of 18 psi and at average temperatures
between 50
C and 60
C.
barriers of the atmosphere when the temperature and
pressure are at an ideal condition for cloud formation. It also
shows the different types of clouds formed at the different
levels. Note: Clouds are formed mostly of water crystals,
but can also contain particles of rock and dust.
14. According to Figure 1, as pressure within the atmo-
spheric layers increases, temperature within the atmo-
spheric layers:
F. increases only.
G. decreases only.
H. increases up to 6 psi, then decreases.
J. decreases up to 10 psi, then increases.
15. According to the information given in Figure 1, clouds
within the stratosphere are most likely formed:
A. under a pressure of 4 psi and 20
C.
B. under a pressure of 10 psi and 30
C.
C. over a pressure of 12 psi and 40
C.
D. over a pressure of 14 psi and 50
C.
16. If a pressure of 7 psi were sustained within the atmo-
sphere, according to Figure 1, which of the following
types of clouds would likely form?
F. Cirrus
G. Cumulus
H. Nimbus
J. Stratus
GO ON TO THE NEXT PAGE.
PRACTICE TEST 2 SCIENCE REASONING TEST 137
4
) ) ) ) ) ) ) ) )
4
PASSAGE IV
Because fish live in water they are exposed to any bacteria
that exist in the water. Table 1 lists the habitat choices of 7
species of fish in a local pond and the fish’s ability to combat
the effects of the bacteria found in the water.
Table 1
Fish species Relative ability to
combat bacteria
Habitat Exposure to
waterborne bacteria
A <0.2 Shallow water with plants None
B <0.3 Shallow water with no plants Low
C 0.2 Shallow water with no plants Low
D 0.3 Deep water with no plants Moderate
E 0.4 Shallow water with plants High
F 0.6 Shallow water with plants High
G 1.3 Shallow water with plants High
Figure 1 shows the percent of fish that survive
to adulthood in the lab for the 7 species, after exposure
to water with bacteria present or exposure to water with the
bacteria removed.
Figure 1
Figure 2 shows predicted bacteria levels over time in
4 geographic regions with fish populations.
Figure 2
GO ON TO THE NEXT PAGE.
138 PRACTICE TEST 2 SCIENCE REASONING TEST
4
) ) ) ) ) ) ) ) )
4
17. Based on the information in Figure 1, fish from which
species are most likely to survive prolonged exposure
to bacteria?
A. Species A
B. Species B
C. Species D
D. Species E
18. According to the data in Figure 1, which species showed
the greatest difference between the percent of fish that
survived to adulthood after exposure to unfiltered water,
and the percent of fish that survived to adulthood after
exposure to filtered water?
F. Species A
G. Species C
H. Species E
J. Species G
19. Researchers recently discovered a new species of fish
that lives in deep water without plants. Based on the
data in Table 1, the researchers would predict that
this species’ relative ability to combat bacteria is most
likely:
A. high.
B. moderate.
C. low.
D. nonexistent.
20. According to the information in Table 1, for all the
species shown, as the exposure to bacteria increases,
the relative ability to combat the bacteria generally:
F. decreases only.
G. increases only.
H. decreases, then increases.
J. increases, then decreases.
21. Based on the data in Table 1 and Figure 1, fish that had
the lowest percent of individuals survive to adulthood
when exposed to bacteria tend to:
A. live in shallow water without plants.
B. live in shallow water with plants.
C. live in deep water without plants.
D. live in deep water with plants.
GO ON TO THE NEXT PAGE.
PRACTICE TEST 2 SCIENCE REASONING TEST 139
4
) ) ) ) ) ) ) ) )
4
PASSAGE V
While digging in a remote site in Africa, paleontologists
discovered a collection of fossilized dinosaur bones. The
bones were dated back to the Jurassic period, and have been
confirmed to be from a dinosaur known as a velociraptor.
Two paleontologists discuss the finding.
Paleontologist 1
Once the well-preserved bones are assembled it is clear
that they are velociraptor bones from the Jurassic period.
The bones are long in the arms, indicating that the veloci-
raptor was definitely capable of flight. You can see that there
are cuts within the arm/wing bones of this dinosaur, indi-
cating that it was caught while in flight. Perhaps it was
attempting an escape from a more predatory dinosaur, such
as tyrannosaurus rex. It is obvious from the body struc-
ture of the velociraptor that it was an effective hunter and
predator. It was most likely quick to swoop in on its prey
and was more than able to carry the prey away on its own.
The form and function of the velociraptor has been mis-
understood until this important discovery. The condition of
these bones offers a clear picture of the way in which the
velociraptor lived.
Paleontologist 2
Indeed, the velociraptor bones are in excellent condition.
The long arm bones are indicative of the dinosaurs abil-
ity to scavenge prey and fend off larger predators. The cuts
within the arm bones show that the velociraptor often stole
its meals—the marks resemble defense wounds, perhaps
from forcing other would-be scavengers away from the free
meal. The structure of the velociraptors feet indicates that
it was a fast runner and was able to maneuver well through
the high trees and undergrowth. This would certainly have
allowed the velociraptor to quickly escape predators and
possibly arrive at a kill-site before other larger dinosaurs,
such as tyrannosaurus rex, descended upon the leftovers.
The bones that were discovered answer many questions
about the velociraptor, but they also bring up many new
issues to consider.
22. Paleontologist 1’s viewpoint contains the basic
assumption that the velociraptor must have been:
F. unknown until the discovery of these bones.
G. an ineffective hunter.
H. previously mischaracterized.
J. unable to escape large predators.
23. Paleontologist 1 would most likely state that the cuts on
the velociraptor bones were the result of:
A. failed attempts to fly.
B. fending off a competing scavenger.
C. an attack by a larger predator.
D. mistakes made in assembling the bones.
24. Suppose that the fossilized remains of another dinosaur
species with long arm bones were discovered, and
scientists determined that this dinosaur lived at the
same time as the velociraptor. According to the passage,
Paleontologist 2 would most likely conclude that:
F. the new dinosaur could fly.
G. the new dinosaur could be a scavenger.
H. the new dinosaur could not escape from predators.
J. the new dinosaur could swoop in on its prey.
25. Paleontologist 2’s viewpoint regarding the velociraptor
as a scavenger was based on the dinosaurs:
A. strong musculature.
B. excellent condition.
C. long arm bones.
D. ability to fly.
26. Paleontologist 1 would most likely support which of
the following statements about the lifestyle of the
velociraptor?
F. The velociraptor was a predatory dinosaur capable
of flight, and is only now being understood.
G. The velociraptor was a dinosaur who scavenged
other dinosaurs’ kills.
H. The velociraptor was a fast runner that could easily
out-maneuver its predators in order to survive.
J. The velociraptor was hunted by many other
dinosaurs during its time on Earth.
27. Assuming all are true, both paleontologists would
most likely agree with which of the following facts
concerning the velociraptor?
A. It was threatened by larger dinosaurs, such as
tyrannosaurus rex.
B. It
was
unable to sustain flight.
C. It was not built for speed, and therefore, could not
easily fend for itself.
D. It was not an effective hunter.
28. Both Paleontologists 1 and 2 would most likely agree
with which of the following statements about the
discovery of the velociraptor bones? The bones:
F. did not clarify any assumptions about the velocirap-
tor.
G. provided some useful information regarding the
velociraptor.
H. could not be assembled properly due to the poor
condition in which they were found.
J. completely altered both paleontologist’s viewpoints
regarding the velociraptor.
GO ON TO THE NEXT PAGE.
140 PRACTICE TEST 2 SCIENCE REASONING TEST
4
) ) ) ) ) ) ) ) )
4
PASSAGE VI
The peaks of mountains often lose sediment due to
wind erosion. Figure 1 shows mountain peak compositions,
mountain heights, in meters (m), and the net change in
meters (m), in mean peak height (MPH) from 1910 to 1970
along a section of the Rocky Mountains. A net negative
change in MPH indicates a net loss of sediment and a net
positive change in MPH indicates a gain of sediment.
Figure 1
Table 1 shows the percentage of a year that horizontal
sections of a mountain are exposed to wind.
Table 1
Peak section
height (m)
Percentage of the year that peak
section is exposed to wind
0.0–0.5 1.1
0.5–1.0 3.1
1.0–1.5 7.2
1.5–2.0 10.5
2.0–2.5 14.2
2.5–3.0 19.4
3.0–3.5 23.7
3.5–4.0 29.3
4.0–4.5 37.4
4.5–5.0 42.3
5.0–5.5 48.0
Note: Heights are measured from mean (average)
sea level.
Figure 2 shows Peak C and D erosion rates, in m/y, as they
relate to percentage of a year that mountain peak section is
exposed to wind.
Figure 2
29. According to Figure 1, at a distance of 9 km along the
mountain range, peaks of what composition are present,
if any?
A. Peaks of slate
B. Peaks of shale
C. Peaks of limestone
D. No peaks are present
30. According to the information in Figure 1, which of the
following properties was used to distinguish the various
materials that compose the peaks in the study area?
F. Particle size
G. Particle clarity
H. Particle color
J. Particle density
31. Based on the information listed in Table 1, a peak sec-
tion with a height of 5.5–6.0 m would be exposed to
wind approximately what percentage of a year?
A. 22%
B. 39%
C. 48%
D. 53%
32. According to Figures 1 and 2, the difference between
Peak C and Peak D erosion rates could best be explained
as a difference in the:
F. heights of the two peaks.
G. force of the winds on the two peaks.
H. composition of the two peaks.
J. annual snowfall on the two peaks.
GO ON TO THE NEXT PAGE.
PRACTICE TEST 2 SCIENCE REASONING TEST 141
4
) ) ) ) ) ) ) ) )
4
33. According to Table 1, which of the following figures
best represents the relationship between the height of
a peak section and the percentage of a year that peak
section is exposed to wind erosion?
34. According to information in the passage, wind erosion
often results in:
F. an increase in the percentage of a mountain peak that
is exposed to snow.
G. a reduction in the overall surface area of mountain
peaks.
H. a higher number of slate and shale deposits on
mountain peaks.
J. a lower number of record snowfalls each year.
GO ON TO THE NEXT PAGE.
142 PRACTICE TEST 2 SCIENCE REASONING TEST
4
) ) ) ) ) ) ) ) )
4
PASSAGE VII
A biologist investigated some of the environmental factors
that could influence the growth of certain types of bacteria.
The following experiments were conducted at a constant
temperature, and no sample was tested more than once.
Experiment 1
Ten samples of bacteria were placed in each of 2 Petri
dishes, the bottoms of which were each half moist and half
dry. The dishes were covered with Petri dish lids. Dish 1
was placed in a darkened area and Dish 2 was placed in a
lighted area. After 2 hours the location of bacterial growth
in each dish was recorded (Table 1).
Table 1
Dry side Moist side
Dish 1 (in dark) 1 9
Dish 2 (in light) 2 8
Experiment 2
Ten samples of bacteria were placed in each of 2 Petri
dishes. The dishes were covered with Petri dish lids. Dish 1
was placed in a darkened area and Dish 2 was placed directly
under a 25-watt incandescent lamp, creating a warm, lighted
environment. After 2 hours the amount of bacterial growth
in each dish was recorded and compared to the amount of
growth in a control sample that was placed in a Petri dish
and left in a regularly lighted area (Table 2).
Table 2
Growth proportional to control
Dish 1 (in dark) 0.93
Dish 2 (under lamp) 1.06
Experiment 3
Ten samples of bacteria were placed in each of 2 Petri
dishes. Four different environments were created in each
dish—dry/lighted, dry/dark, moist/lighted, and moist/dark.
The bottoms of the Petri dishes were each half moist and half
dry. The dishes were covered with Petri dish lids. Dish 1 was
placed in a darkened area and Dish 2 was placed directly
under a 25-watt incandescent lamp, creating a warm, lighted
environment. After 2 hours the amount of bacterial growth
in each dish was recorded and compared to the amount of
growth in a control sample that was placed in a Petri dish
and left in a regularly lighted area (Table 3).
Table 3
Growth proportional to control
Moist side Dry side
Dish 1 (in dark) 0.99 0.53
Dish 2 (under lamp) 1.15 0.67
35. One reason refrigeration might be used as a means to
control bacteria growth is that bacteria:
A. grow at a faster rate in warm environments.
B. grow at a slower rate in warm environments.
C. require good ventilation.
D. prefer dry environments.
36. Based on the results of Experiment 3, the greatest
proportional growth was observed:
F. on the moist side of Dish 1.
G. on the moist side of Dish 2.
H. on the dry side of Dish 1.
J. on the dry side of Dish 2.
37. Which of the following conclusions is supported by the
results of Experiment 1?
A. Bacteria prefer light environments to dark environ-
ments.
B. Bacteria exhibit an equal preference for light and
dark environments.
C. Bacteria prefer moist environments to dry environ-
ments, regardless of lighting conditions.
D. Bacteria exhibit an equal preference for dry and
moist environments.
38. One criticism of these experiments might be that the
presence of more than one sample of bacteria in each
Petri dish might have had an effect on the results. Which
of the following changes in experimental design could
be made to counter this criticism?
F. Use additional species of bacteria in each test.
G. Use only bacteria that was taken directly from
nature and not generated in a lab.
H. Place each sample in a separate Petri dish.
J. Vary the size of the starting sample.
GO ON TO THE NEXT PAGE.
PRACTICE TEST 2 SCIENCE REASONING TEST 143
4
) ) ) ) ) ) ) ) )
4
39. Bacteria are known to exist on nearly every surface
of the world. On the basis of the experimental results,
which of the following environments would provide the
conditions best suited for a high growth rate?
A. The surface of a desert rock.
B. The bottom of a Great Lake.
C. The surface of Antarctic ice sheet.
D. Beneath a rock in a tropical forest.
END OF THE SCIENCE REASONING TEST.
STOP! IF YOU HAVE TIME LEFT OVER, CHECK YOUR WORK ON THIS SECTION ONLY.
40. In the 3 experiments, the environmental factors that
could influence growth were evaluated by recording
data about growth after 2 hours. Because bacteria dou-
ble population size in short intervals, better information
about growth might be achieved by recording data:
F. after 10 minutes.
G. at 30-minute intervals for 1 hour.
H. after 1 hour.
J. at 10-minute intervals for 2 hours.
144 PRACTICE TEST 2 WRITING TEST PROMPT
WRITING TEST PROMPT
DIRECTIONS: This test is designed to assess your writing skills. You have 30 minutes to plan and write
an essay based on the stimulus provided. Be sure to take a position on the issue and support your
position using logical reasoning and relevant examples. Organize your ideas in a focused and logical
way, and use the English language to clearly and effectively express your position.
When you have finished writing, refer to the Scoring Rubrics discussed in the Introduction (page 4)
to estimate your score.
Some high schools ban students from driving to and from
school if they live in an area with bus service. Adminis-
trators think this will reduce morning and afternoon traffic
accidents and congestions as well as alleviate morning tar-
diness. Opponents say that a student with a driver’s license
should have the same right to drive to school as do faculty
and staff with licenses. Some students say that while they
are technically inside the boundary for bus service, walking
to and from the bus stop every day is a major inconvenience.
In your opinion, should high schools ban students’ commut-
ing to reduce traffic and tardiness problems?
In your essay, take a position on this question.You may write
about one of the points of view mentioned above, or you
may give another point of view on this issue. Use specific
examples and reasons for your position.
PRACTICE TEST 2 ANSWER KEY 145
ANSWER KEY
English Test
1. C 21. D 41. D 61. D
2. F 22. J 42. F 62. F
3. A 23. D 43. D 63. C
4. J 24. J 44. G 64. F
5. D 25. A 45. C 65. B
6. H 26. F 46. J 66. G
7. B 27. B 47. C 67. D
8. J 28. H 48. H 68. G
9. D 29. B 49. C 69. B
10. F 30. H 50. F 70. J
11. A 31. B 51. B 71. B
12. G 32. H 52. H 72. J
13. D 33. B 53. B 73. A
14. F 34. F 54. H 74. J
15. B 35. B 55. A 75. D
16. H 36. F 56. G
17. A 37. C 57. D
18. G 38. J 58. G
19. C 39. D 59. C
20. G 40. G 60. H
146 PRACTICE TEST 2 ANSWER KEY
Mathematics Test
1. B 21. B 41. B
2. K 22. F 42. H
3. C 23. B 43. A
4. H 24. J 44. F
5. C 25. A 45. B
6. J 26. G 46. K
7. B 27. B 47. A
8. H 28. G 48. H
9. D 29. E 49. D
10. K 30. G 50. G
11. C 31. C 51. D
12. K 32. F 52. J
13. A 33. C 53. C
14. F 34. G 54. H
15. B 35. B 55. C
16. H 36. K 56. K
17. B 37. A 57. A
18. H 38. F 58. F
19. E 39. A 59. E
20. G 40. G 60. H
PRACTICE TEST 2 ANSWER KEY 147
Reading Test
1. A 21. D
2. H 22. H
3. B 23. A
4. J 24. H
5. C 25. B
6. F 26. H
7. D 27. D
8. G 28. J
9. D 29. A
10. J 30. G
11. B 31. A
12. F 32. H
13. D 33. C
14. H 34. F
15. B 35. A
16. G 36. J
17. C 37. D
18. J 38. H
19. D 39. C
20. F 40. F
Science Reasoning Test
1. C 21. B
2. H 22. H
3. C 23. C
4. G 24. G
5. A 25. C
6. H 26. F
7. C 27. A
8. H 28. G
9. A 29. B
10. F 30. F
11. D 31. D
12. G 32. F
13. A 33. C
14. G 34. G
15. B 35. A
16. J 36. G
17. D 37. C
18. G 38. H
19. B 39. D
20. G 40. J
PRACTICE TEST 2 SCORING GUIDE 149
SCORING GUIDE
Your final reported score is your COMPOSITE SCORE. Your COMPOSITE SCORE is the average of all
of your SCALE SCORES.
Your SCALE SCORES for the four multiple-choice sections are derived from the Scoring Table on
the next page. Use your RAW SCORE, or the number of questions that you answered correctly for each
section, to determine your SCALE SCORE. If you got a RAW SCORE of 60 on the English test, for
example, you correctly answered 60 out of 75 questions.
Step 1 Determine your RAW SCORE for each of the four multiple-choice sections:
English
Mathematics
Reading
Science Reasoning
The following Raw Score Table shows the total possible points for each section.
RAW SCORE TABLE
KNOWLEDGE AND SKILL AREAS RAW SCORES
ENGLISH 75
MATHEMATICS 60
READING 40
SCIENCE REASONING 40
WRITING 12
150 PRACTICE TEST 2 SCORING GUIDE
Multiple-Choice Scoring Worksheet
Step 2 Determine your SCALE SCORE for each of the four multiple-choice sections using the
following Scoring Worksheet. Each SCALE SCORE should be rounded to the nearest number according
to normal rules. For example, 31.2 31 and 31.5 32. If you answered 61 questions correctly on the
English section, for example, your SCALE SCORE would be 29.
English × 36 = ÷ 75 =
RAW SCORE 2 (*correction factor)
SCALE SCORE
Mathematics × 36 = ÷ 60 =
RAW SCORE + 1 (*correction factor)
SCALE SCORE
Reading × 36 = ÷ 40 =
RAW SCORE + 2 (*correction factor)
SCALE SCORE
Science Reasoning × 36 = ÷ 40 =
RAW SCORE + 1.5 (*correction factor)
SCALE SCORE
*The correction factor is an approximation based on the average from several recent ACT tests. It is most valid for scores in
the middle 50% (approximately 16–24 scale composite score) of the scoring range.
The scores are all approximate. Actual ACT scoring scales vary from one administration to the next based upon several factors.
If you take the optional Writing Test, you will need to combine your English and Writing scores to
obtain your final COMPOSITE SCORE. Once you have determined a score for your essay out of 12 possible
points, you will need to determine your ENGLISH/WRITING SCALE SCORE, using both your ENGLISH
SCALE SCORE and your WRITING TEST SCORE. The combination of the two scores will give you an
ENGLISH/WRITING SCALE SCORE, from 1 to 36, that will be used to determine your COMPOSITE
SCORE mentioned earlier.
Using the English/Writing Scoring Table, find your ENGLISH SCALE SCORE on the left or right hand
side of the table and your WRITING TEST SCORE on the top of the table. Follow your ENGLISH SCALE
SCORE over and your WRITING TEST SCORE down until the two columns meet at a number. This
number is your ENGLISH/WRITING SCALE SCORE and will be used to determine your COMPOSITE
SCORE.
Step 3 Determine your ENGLISH/WRITING SCALE SCORE using the English/Writing Scoring Table
on the following page:
English
Writing
English/Writing
PRACTICE TEST 2 SCORING GUIDE 151
ENGLISH/WRITING SCORING TABLE
ENGLISH WRITING TEST SCORE ENGLISH
SCALE
2 3 4 5 6 7 8 9 10 11 12
SCALE
SCORE SCORE
36 26 27 28 29 30 31 32 33 34 32 36 36
35 26 27 28 29 30 31 31 32 33 34 35 35
34 25 26 27 28 29 30 31 32 33 34 35 34
33 24 25 26 27 28 29 30 31 32 33 34 33
32 24 25 25 26 27 28 29 30 31 32 33 32
31 23 24 25 26 27 28 29 30 30 31 32 31
30 22 23 24 25 26 27 28 29 30 31 32 30
29 21 22 23 24 25 26 27 28 29 30 31 29
28 21 22 23 24 24 25 26 27 28 29 30 28
27 20 21 22 23 24 25 26 27 28 28 29 27
26 19 20 21 22 23 24 25 26 27 28 29 26
25 18 19 20 21 22 23 24 25 26 27 28 25
24 18 19 20 21 22 23 23 24 25 26 27 24
23 17 18 19 20 21 22 23 24 25 26 27 23
22 16 17 18 19 20 21 22 23 24 25 26 22
21 16 17 17 18 19 20 21 22 23 24 25 21
20 15 16 17 18 19 20 21 21 22 23 24 20
19 14 15 16 17 18 19 20 21 22 23 24 19
18 13 14 15 16 17 18 19 20 21 22 23 18
17 13 14 15 16 16 17 18 19 20 21 22 17
16 12 13 14 15 16 17 18 19 20 20 21 16
15 11 12 13 14 15 16 17 18 19 20 21 15
14 10 11 12 13 14 15 16 17 18 19 20 14
13 10 11 12 13 14 14 15 16 17 18 19 13
12 9 10 11 12 13 14 15 16 17 18 19 12
11 8 9 10 11 12 13 14 15 16 17 18 11
10 8 9 9 10 11 12 13 14 15 16 17 10
9 7 8 9 10 11 12 13 13 14 15 16 9
8 6 7 8 9 10 11 12 13 14 15 16 8
7 5 6 7 8 9 10 11 12 13 14 15 7
6 5 6 7 7 8 9 10 11 12 13 14 6
5 4 5 6 7 8 9 10 11 12 12 13 5
4 3 4 5 6 7 8 9 10 11 12 13 4
3 2 3 4 5 6 7 8 9 10 11 12 3
2 2 3 4 5 6 6 7 8 9 10 11 2
1 1 2 3 4 5 6 7 8 9 10 11 1
152 PRACTICE TEST 2 SCORING GUIDE
Step 4 Determine your COMPOSITE SCORE by finding the sum of all your SCALE SCORES for each of
the four sections: English only (if you do not choose to take the optional Writing Test) or English/Writing
(if you choose to take the optional Writing Test), Math, Reading, and Science Reasoning, and divide by 4
to find the average. Round your COMPOSITE SCORE according to normal rules. For example, 31.2 31
and 31.5 32.
+ + + =
ENGLISH OR MATHEMATICS READING SCIENCE SCALE SCORE
ENGLISH/WRITING SCALE SCORE SCALE SCORE SCALE SCORE TOTAL
SCALE SCORE
÷ 4 =
SCALE SCORE TOTAL COMPOSITE SCORE
PRACTICE TEST 2 ANSWERS AND EXPLANATIONS 153
ANSWERS AND EXPLANATIONS
English Test Explanations
PASSAGE I
1. The best answer is C. The narrator was chosen
“last spring,” which was in the past. Answer choice
A is incorrect because the moment the narrator is
referring to is relative to the time the narrator wrote
the passage, not another time in the past. Answer
choices B and D can be eliminated because they
are not past tense.
2. The best answer is F. The sentence appropriately
uses the relative pronoun who to introduce the
clause that modifies couple; this sentence is correct
as it is written. The pronoun who also functions as
the subject of the clause.
3. The best answer is A. The sentence represents
a clear, complete thought that is grammatically
correct. It is correct to begin a new sentence
with speaking. You can eliminate the other answer
choices because either they create incomplete
sentences (B), or are otherwise grammatically
incorrect.
4. The best answer is J. The sentence indicates that
eating breakfast with the host family was a routine
action in the past. Answer choice J is best because
it includes would, which suggests repeated eating
of breakfast with the couple. You can eliminate
answer choice F because it is written in the present
tense.
5. The best answer is D. The two clauses are unre-
lated and, therefore, you can connect the two
separate ideas by using the word then. Answer
choices B and C can be eliminated because they
express a cause-and-effect relationship that does
not fit with the sentence.
6. The best answer is H. This question asks you to
place the apostrophe correctly in the underlined
portion of the sentence. Answer choice H is best
because the extended family is that of both parents
together. This is made even clearer by the subject
they in the following sentence.
7. The best answer is B. Answer choice B correctly
uses the infinitive form to tend to explain why
the host couple must travel. Choice C is incor-
rect because inflected forms like tended require an
expressed subject. Choice D is incorrect because
the to that follows is realized as a preposition linked
to travel. The couple is not physically traveling to
the business.
8. The best answer is J. This question tests your
ability to spot relevance. Neither the underlined
portion nor the information is answer choice G and
H add anything relevant to the sentence. Therefore,
it would be best to omit the underlined portion and
simply end the sentence with “another.”
9. The best answer is D. Logically, the question is
asking whether the speaker would like to live with
someone who speaks English or someone who does
not speak English. The term whether only requires
one side of a two-sided situation (e.g. “I don’t know
whether she has a roommate,” not “I don’t know
whether she has a roommate or lives by herself.”)
10. The best answer is F. The word whom is an object
pronoun, meaning it will occur in object, not sub-
ject, position. In this case, it is the object of find.
Remember, whom refers to Paolo, the logical direct
object. This becomes clear if you reorder the clause:
“I was surprised to find Paolo playing one of my
favorite CDs on the stereo!”
11. The best answer is A. The essay maintains a pos-
itive, uplifting tone with regard to the speakers
friendship with Paolo. The other answer choices
do not match the tone of the essay.
12. The best answer is G. The word talk takes a
prepositional object starting with about; you can-
not divide the phrase with a comma. Answer choice
H is incorrect because modifiers like “my Brazilian
friend” that come before proper names do not need
a comma.
13. The best answer is D. The sentence is used to
describe a faulty or incomplete first impression,
then uses but to introduce a revision to it. Such
clauses introduced by a subordinating conjunction
are offset from the first clause of the sentence with
a comma.
14. The best answer is F. The speaker is summa-
rizing his trip into one important lesson. Answer
choices G and H do not represent how meaningful
the
lesson
is to the speaker and answer choice J is
awkward as a modifier of lesson.
154 PRACTICE TEST 2 ANSWERS AND EXPLANATIONS
15. The best answer is B. The essay describes an
enjoyable friendship between people of two dif-
ferent nationalities that begins in the context of
a foreign culture. Answer choices A and C have
a negative tone that does not match the rest of
the essay. Answer choice D can be eliminated
because it is not relevant to the passage.
PASSAGE II
16. The best answer is H. The clause subordinate to
apartment is I’m renting and cannot be divided by
a comma. The sentence up to the word renting is an
adverbial, describing the location of the restaurant,
so it must be followed by a comma.
17. The best answer is A. The paragraph is written
in present tense. Answer choices B and C are past
tense, and imply that the restaurant is not located
there anymore. Answer choice D implies that the
restaurant is not located there yet.
18. The best answer is G. The prepositional phrase
introduced by in modifies fountain and cannot be
divided by a comma. Answer choice J can be
eliminated because a semicolon should be used to
separate two independent clauses.
19. The best answer is C. Adjectives such as sinful
modify nouns, such as deliciousness, which can
be modified by prepositional phrases, such as of
Joe’s Special Reuben. The other answer choices
incorrectly pair adjectives, adverbs, and nouns.
20. The best answer is G. This question tests your
ability to recognize redundancy in a sentence. New-
comers implies people who have never seen his
creations. The other answer choices are redundant.
21. The best answer is D. This question requires you
to determine the correct punctuation. No punctu-
ation is necessary between nouns like sauerkraut
and gerunds like spilling, which modify them.
22. The best answer is J. Sentence 4 is about the menu
in the window. It should be placed after the sen-
tence that describes the menu. Placing it anywhere
else in the paragraph would cause the paragraph
not to make sense.
23. The best answer is D. This answer choice
describes how the unwelcoming appearance of
the restaurant does not reveal the truth about the
delicious food. The other answer choices can be
eliminated because they are not relevant to the topic
of the paragraph.
24. The best answer is J. Belies is a transitive verb that
takes what follows it as its direct object. A comma
cannot be placed after belies; therefore, answer
choices F and H can be eliminated. Answer choice
G would create a sentence fragment, and can be
eliminated.
25. The best answer is A. The verbs must agree with
the subject refrigerator. Answer choice D is not
appropriate when describing a single refrigerator
case, and can be eliminated. Answer choice B does
not make sense. Answer choice C is not inflected
for tense.
26. The best answer is F. This sentence provides a
logical transition between the previous sentence,
which is about building repair, and the following
sentence, which is about how the food is enough to
sustain the charm of the restaurant.
27. The best answer is B. Many is the appropriate
adjective for “A lot of people” from the previous
sentence. It emphasizes the disparity between the
narrators knowledge of regular customers’ faces
and the sandwiches they eat. Answer choice A can
be eliminated because while much can apply to
mass nouns (e.g. much trash, much noise), it can-
not be used with countable nouns. For those, many
must be used (e.g. many people, many marbles).
Answer choice C is an adverb and would, there-
fore, be ungrammatical. Answer choice D does not
make sense in context.
28. The best answer is H. The phrase describes the
classics, which by itself may be unclear. Answer
choice G can be eliminated because the information
is important to your knowledge of what the classics
are. Answer choices F and J are not supported by
the passage.
29. The best answer is B. The information is made
clear without the use of excess commas or weak
words like makes and
this place.
The other answer
choices are too wordy or have awkward pauses.
30. The best answer is H. This question requires you
to determine the correct punctuation. No punc-
tuation is required between two noun phrases
conjoined with and.
PASSAGE III
31. The best answer is B. The speaker is talking about
Native Americans in the past; therefore, answer
choices A, C, and D may be eliminated.
32. The best answer is H. A list of only two items does
not need punctuation to separate them; therefore,
the other answer choices may be eliminated.
PRACTICE TEST 2 ANSWERS AND EXPLANATIONS 155
33. The best answer is B. The speaker is talking about
the ferries in past tense; therefore, the other answer
choices may be eliminated.
34. The best answer is F. The phrase as it is written is
clear and concise. Answer choice G is wordy and
answer choices H and J do not make sense in the
sentence.
35. The best answer is B. The underlined sentence
explains that a potential bridge was reasonable
because the Brooklyn Bridge was a success. The
other answer choices are not supported by the
passage.
36. The best answer is F. To “take action on” some-
thing is a common idiomatic expression; therefore,
the other answer choices may be eliminated.
37. The best answer is C. The second clause describes
something that stands in opposition to the first
clause; that is, ferry service was resumed, but
something caused it to stop again. The other answer
choices would not link the two clauses correctly.
38. The best answer is J. The possessive form of “it” is
“its,” without the apostrophe; answer choice H may
be eliminated. Here, “its” refers to “ferry service
between peninsulas,” which is singular; therefore,
answer choice G may be eliminated.
39. The best answer is D. The sentence as it is writ-
ten is a fragment. Simply eliminating “that” after
“however corrects this problem. The word “how-
ever,” when used in the middle of a sentence must
be offset with commas.
40. The best answer is G. The preceding sentence
shows the significance of the bridge. Answer
choices H and J can be eliminated because they are
irrelevant to the topic of the essay. Answer choice
F can be eliminated because the sentence does not
explain how the bridge was built.
41. The best answer is D. The elements in the sentence
are ordered logically. The other answer choices sep-
arate elements from each other with use of commas,
which make the sentence less clear.
42. The best answer is F. The other answer choices
would not make sense with the simple present form
“is” used in the sentence.
43. The best answer is D. This question tests your abil-
ity to recognize redundancy. Highway drivers and
travelers in this sentence essentially mean the same
thing. The other answer choices are redundant.
44. The best answer is G. This answer choice does
not fit with the topic of the essay, which is
the Mackinac Bridge in the context of Michigan,
not the context of the all the world’s suspension
bridges.
45. The best answer is C. This essay focuses on
how the bridge came to be built. Answer choices
A and B may be eliminated because the essay
does not describe the entire process of build-
ing the Mackinac Bridge. Answer choice D may
be eliminated because the topic of the essay
is not specifically the reason for building the
bridge.
PASSAGE IV
46. The best answer is J. The underlined sentence dis-
tracts from the intent of the paragraph, and should
be omitted. The other answer choices also include
information that is irrelevant to the topic of the
essay.
47. The best answer is C. The narrator is speaking
about his past. Answer choice A is ungrammatical
and wordy. Answer C differs from B in that it omits
whom, an unnecessary element. Answer D may be
eliminated because the sentence would not make
sense.
48. The best answer is H. This question requires you
to determine the correct punctuation. No punctua-
tion may come between a noun phrase (box in his
room) and the gerund modifying it (brimming).
49. The best answer is C. The simple plural past
tense is correct here. No piece is a singular sub-
ject, and cannot be referred to by answer choice A.
Answer choice B would not make sense in the
sentence.
50. The best answer is F. In Paragraph 2 the author
analyzes the arguments in favor of electronic enter-
tainment and then proposes his counterarguments.
Answer choice G can be eliminated because it
is irrelevant to the authors argument. Answer
choice H is unsupported by the passage and answer
choice J does not provide a logical transition from
the previous paragraph.
51. The best answer is B. In the sentence, but does
not correctly conjoin dynamic and engaging, which
are closely related. The other answer choices are
equivalent to the underlined sentences and are
acceptable.
52. The best answer is H. When placed after Sen-
tence 7, the sentence provides a logical transition
to the subsequent paragraph. Answer choices F and
156 PRACTICE TEST 2 ANSWERS AND EXPLANATIONS
G can be eliminated because their placement would
cause the paragraph to be illogical. The assertion
in answer choice J is false, and may, therefore, be
eliminated.
53. The best answer is B. This question tests your abil-
ity to recognize redundancy in a sentence. Answer
choice B is the best answer because it avoids
redundancy. Answer choices A, C, and D would
introduce redundancy.
54. The best answer is H. The answer choice cor-
rectly modifies weekend. The narrator intends to
describe card shows that occurred multiple times
over multiple weekends. Answer choice F can be
eliminated because it does not accurately convey
what the narrator intends to describe.
55. The best answer is A. The sentence describes a
habitual action in the past. Answer choice C can
be eliminated because it is present tense. Answer
choice B can be eliminated because we does not
make any sense in the sentence.
56. The best answer is G. Adverbials like too are
separated from the main clause by a comma. The
gerund encouraging modifies activity, so too must
be followed by a comma as well.
57. The best answer is D. This answer choice elim-
inates the redundancy. The word screen does not
need to be repeated because the reader already
knows that it is the video screen that silences the
spectators.
58. The best answer is G. This answer choice is con-
sistent with the narrators use of the second person
from the preceding sentence. Answer choices F, H,
and J can be eliminated because they do not use
second person.
59. The best answer is C. The best way to answer this
question is to try the answer choices in the sentence.
Answer choice C is the best because only adverbs
modify adjectives.
60. The best answer is H. The narrator believes
video games do not teach the skills card collecting
teaches. Showing this in the passage indicates the
author finds card collecting superior to electronic
entertainment.
PASSAGE V
61. The best answer is D. This question requires you to
determine the correct punctuation. In the sentence,
wait takes a prepositional object headed by until,
so punctuation must not separate them.
62. The best answer is F. The clause headed by which
provides descriptive detail about why spyware is
a threat. The remaining answer choices are not
grammatical.
63. The best answer is C. The writer sets the computer
virus in opposition to spyware, to show how the
two are unlike each other. The other answer choices
compare spyware with a computer virus, which is
not the intent of the writer.
64. The best answer is F. The best way to answer
this question is to try the answer choices in
the sentence. Here, often coordinates well with
usually. Answer choice H can be eliminated
because always does not make sense with but. The
other answer choices would not make sense in
context.
65. The best answer is B. However, when used in the
middle of a clause is set apart with commas. The
other answer choices can be eliminated because
they don’t follow this rule.
66. The best answer is G. The paragraph describes
some harmful spyware programs. This sentence
effectively introduces the topic and links Para-
graphs 2 and 3.
67. The best answer is D. This is clear, concise, and in
active voice. The other answer choices are wordy.
68. The best answer is G. Answer choice G is the
only choice that avoids redundancy. Detection util-
ities suggest that the utilities detect spyware. It is
unnecessary to restate the fact that spyware is being
detected.
69. The best answer is B. The verbs must agree with
the plural subject Detection utilities. Answer choice
B is the only answer in which scan and remove
correspond with the plural subject.
70. The best answer is J. The writer is using present
tense. Answer choices G may be eliminated
because it is in past tense. Computer is the logi-
cal object of protect, and therefore, answer choices
H and F can be eliminated.
71. The best answer is B. This is the only answer
choice that provides a specific sound web surfing
habit. Answer choice A is not specific enough to
provide you with an example of a habit. Answer
choices C and D are irrelevant to the argument the
writer is making.
72. The best answer is J. Negative elements appear at
the beginning of imperative clauses. Never is neg-
ative, and therefore, would fit best at the beginning
of the sentence.
PRACTICE TEST 2 ANSWERS AND EXPLANATIONS 157
73. The best answer is A. The sentence begins with an
“If clause and ends with a clause describing the
result. These two must be separated by a comma
and the subject must be repeated. The other answer
choices may, therefore, be eliminated.
74. The best answer is J. The sentence would provide
a logical conclusion to the essay. Answer choices F,
G, and H can be eliminated because if the sentence
were placed in one of those paragraphs, it would
not support the arguments it followed.
75. The best answer is D. Though the author might
know how to protect computers from spyware,
he makes no reference to its programming or the
ethical issues that surround it.
158 PRACTICE TEST 2 ANSWERS AND EXPLANATIONS
Mathematics Test Explanations
1. The correct answer is B. To find the midpoint
of two points, you can take the average of the
x and y coordinates. If point X has coordinates
(4,0) and point Y has coordinates (0,8), then
the midpoint is:
(4 + 0)
2
,
(0 + 8)
2
!
4
2
,
8
2
!
(2, 4)
2. The correct answer is K. To solve, use the
Pythagorean Theorem (c
2
= a
2
+b
2
). Using values
from MNO, you can set up the equation like this:
10
2
= 6
2
+ NO
2
100 = 36 + NO
2
NO
2
= 100 36 = 64
NO =
64 = 8
3. The correct answer is C. Since a distance in
meters, M, can be approximated by multiplying
a distance in yards, Y , by 1.0936, it follows that
M Y (1.0936).
4. The correct answer is H. Because Seth has 4 plaid
shirts and 5 solid-colored shirts, the total number
of shirts is 4 +5 = 9. Of these 9 shirts, 4 are plaid.
Thus the probability that a randomly selected shirt
will be plaid is
4
9
.
5. The correct answer is C. Unless otherwise spec-
ified, average means “arithmetic mean,” which
is defined as the sum of a set of values divided
by the number of values. Therefore, you can see
that the average number of enrollments per day is
(17 + 19 + 23 + 14 + 25 + 28)
6
=
126
6
, or 21.
6. The correct answer is J. When parallel lines are
cut by a transversal (such as segment PR in this
problem), “alternate interior angles” are congruent.
In this problem, y and 4 are alternate inte-
rior angles, and so you can conclude that they are
congruent.
7. The correct answer is B. The easiest way to solve
this problem is to take 25% of the original price
and deduct it from the original price. To find 25%
of the sale price of the carton of paper, you
would multiply $27.00 by 25%, or 0.25. Therefore,
the sale price of the carton of paper would
be $27.00 ($27.00)(0.25) = $27.00 $6.75, or
$20.25.
8. The correct answer is H. To solve this problem,
you must recognize that parallel lines always have
the same slope. Remember that to find the slope of
the line, you have to convert the equation 2x3y =
7 into slope-intercept form (y = mx + b, where m
is the slope):
2x 3y 2x = 7 2x
(3y)
3
=
(2x + 7)
(3)
y =
2x
3
+
7
3
y =
2
3
x
7
3
Thus the slope of this line, or any line parallel to
it, is
2
3
.
9. The correct answer is D. In order to solve this
problem, you must realize that if Andrew had
$28,000 remaining after paying 30% in taxes, then
the $28,000 constitutes 100% 30% or 70% of the
original prize, P. Therefore, 0.7P = 28,000. Divid-
ing by 0.7, you can conclude that the original cash
value of the prize was P =
28,000
0.7
, or $40,000.
10. The correct answer is K. To solve this problem, it
is useful to assign values to the number of apples
that Melissa and Marcia both possess. If Marcia
has 10 apples, Melissa has 10 3 = 7 apples. If
Melissa gives 2 of her 7 apples to Marcia, Melissa
is left with 72 = 5 apples. When Marcia receives
2 more apples, she has 10 + 2 = 12 apples. Since
Marcia now has 12 apples and Melissa now has
5 apples, Melissa has 12 5 = 7 fewer apples than
Marcia.
11. The correct answer is C. The absolute value of
a number is its distance from zero, regardless of
whether it is positive or negative. Therefore, the
value of |5 9| = | 4| = 4.
12. The correct answer is K. Since this problem
requires you to multiply two binomials, you can uti-
lize the FOIL (First, Outside, Inside, Last) method
PRACTICE TEST 2 ANSWERS AND EXPLANATIONS 159
to multiply the expressions.
First: (3m)(m
2
) = 3m
3
Outside: (3m)(n) = 3mn
Inside: (n)(m
2
) = m
2
n
Last: (n)(n) = n
2
Finally, add all these terms up to come up with your
final answer. (3m + n)(m
2
n) = 3m
3
3mn +
m
2
n n
2
.
13. The correct answer is A. To solve this problem,
you must distribute and add like terms, as follows:
13 2(x + 5) =
13 2x 10 = 2x + 3
14. The correct answer is F. Remember that the rule
for exponents states that for base number b and
exponents x and y, (b
x
)
y
= b
xy
. Thus, when you
apply the numbers from this problem, you find that
(n
7
)
11
= n
(7)(11)
= n
77
.
15. The correct answer is B. To solve this problem,
recognize that the repeating decimal has four places
(0.3456), and that the fourth place is occupied by
the number 6. Therefore, every place that is a mul-
tiple of 4 will be represented by the number 6.
Since 217 is not divisible by 4, you know that the
217th digit cannot be 6; eliminate answer choice
E. Because 216 is a multiple of 4, the 216th digit
will be 6. Therefore, the 217th digit must be 3, the
next digit in the repeating decimal.
16. The correct answer is H. If a square has side x,
then its perimeter is 4x; this is because a square is
defined as a rectangle where all four sides are of
equal length. Since the perimeter of the square is
48, then 48 = 4x and x =
48
4
= 12. Thus, the
length of one side of the square is 12. The area of
a square is defined as (side)
2
; therefore the area of
this square is 12
2
or 144.
17. The correct answer is B. The easiest way to
solve this problem is to remember that when two
binomial expressions are multiplied, there is a
predictable result. Take the following generalized
example: (x + a)(x b) = x
2
bx + ax ab.
If x
2
bx + ax ab = 0, then the solutions to
the equation are x = a and x = b. The prod-
uct of the solutions is ab. With this expression,
x
2
+3x21 = 0, the product of the solutions (ab)
is 21.
18. The correct answer is H. Remember that a differ-
ence of squares factors easily, such as: a
2
b
2
=
(a + b)(a b). Using the same technique, you can
factor a
16
16 into (a
8
+ 4)(a
8
4). The fac-
tor (a
8
4) is another difference of squares, so
it can be factored further into itself: (a
8
4) =
(a
4
+ 2)(a
4
2). Of these factors, only (a
4
+ 2) is
an answer choice.
19. The correct answer is E. Recall that dividing by
a fraction is equivalent to multiplying by the recip-
rocal. When
1
4
is substituted for n in the following
expression,
2n 5
n
, the result is:
2
"
1
4
#
5
1
4
=
2
4
5
!
4
= 2 20 = 18
20. The correct answer is G. Since 90 minutes is
equal to 1.5 hours, a proofreader who can read
40 pages in one hour can read (1.5)(40) or 60 pages
in 1.5 hours.
21. The correct answer is B. The height, h, can
be found using the Pythagorean Theorem (c
2
=
a
2
+ b
2
):
5
2
= 3
2
+ h
2
25 = 9 + h
2
h
2
= 16, or h = 4.
Thus, when you multiply the base of the parallelo-
gram by its height, the area of the parallelogram is
9 × 4 = 36.
22. The correct answer is F. For a certain quadratic
equation ax
2
+ bx + c = 0, if x =
a
b
is a solution,
then a possible factor would be (bx a). Since two
solutions for ax
2
+ bx + c = 0 are x =
3
4
and
x =
2
5
, then possible factors are (4x 3) and
(5x + 2).
23. The correct answer is B. The diagonals of a rhom-
bus intersect at their midpoints and form right
angles as shown below.
160 PRACTICE TEST 2 ANSWERS AND EXPLANATIONS
Since the diagonals meet at their midpoints and
form right angles, they form a right triangle with
legs
12
2
= 6 and
32
2
= 16. To find the length
of a side of the rhombus, you can simply use the
Pythagorean Theorem and solve where the side of
the rhombus, s, is the hypotenuse: s
2
= 6
2
+16
2
=
292; s is approximately equal to 17.09.
24. The correct answer is J. If a rectangular parking
lot has a length, l, that is 3 feet longer than its
width, w, then l = 3 +w, or w = l 3. The area of
a rectangle is equal to its length times it width, or
A = lw. Since the area of this parking lot is 550,
lw = 550. Substituting (l 3) for
550 = l(l 3) =
550 = l
2
3l
l
2
3l 550 = 0.
To solve for l, factor the quadratic equation to get
(l + 22)(l 25) = 0, making l = 22 or l = 25.
Since negative values for length do not make sense
in this context, the length is 25.
25. The correct answer is A. To find the slope of the
line between any two points (x
1
, y
1
) and (x
2
, y
2
),
you can use the equation
(y
2
y
1
)
(x
2
x
1
)
. Therefore,
when you have the points (3,7) and (4,8) it fol-
lows that the slope of the line joining these points
is
(8 7)
(4 3)
=
15
1
, or 15.
26. The correct answer is G. To find the solution set
of x+2> 4, first solve for x by subtracting 2 from
both sides. The result is x > 6. Thus the solution
set is {x : x > 6).
27. The correct answer is B. To solve this problem,
you need to know that the equation of a circle with
center (h,k) and radius r is (x h)
2
+(y k)
2
= r
2
.
Therefore, the center of the circle in the problem,
(x 3)
2
+ (y + 3)
2
= 4, is (3,3).
28. The correct answer is G. To find the distance
between two points (x
1
, y
1
) and (x
2
, y
2
), you
can use the distance formula, which is d =
$
(x
2
x
1
)
2
+ (y
2
y
1
)
2
. The length of the line
segment that has endpoints (3,4) and (5,6)
will equal the distance between points (3,4) and
(5,6). Therefore, d =
%
(5 (3))
2
+ (6 4)
2
=
$
8
2
+ 10
2
=
64 + 100
=
164
=
$
(4)(41)
=
4
41
= 2
41
29. The correct answer is E. The key to solving this
problem is remembering that the triangle inequality
states that no one side of a triangle can be greater
than the sum of the other two sides. Thus the third
side of the triangle in the problem cannot be greater
than the sum of the other two sides, 4.7 and 9,
which is 13.7. Of the answer choices, only 14 is
too large to be a possible value for the third side of
the triangle.
30. The correct answer is G. To solve this problem,
recall that
n
x
n
y
= n
xy
. Since it is given in the
problem that
n
x
n
y
= n
2
, you can conclude that
n
xy
= n
2
and thus x y = 2.
31. The correct answer is C. To solve, convert the
equation of the line to slope-intercept form (y =
mx + b, where m is the slope and b is the
y-intercept). If 3x + 5y = 8, then:
3x + 5y 3x = 8 3x
(5y)
5
=
(3x + 8)
5
y =
3x
5
+
8
5
Since the equation y =
3x
5
+
8
5
is in slope-
intercept form, the y-intercept is
8
5
.
32. The correct answer is F. To find the cost per
ounce, first convert 3.4 pounds to ounces. Because
there are 16 ounces in a pound, 3.4 pounds is
3.4(16) = 54.4 ounces. To find cost per ounce,
divide the cost in dollars by the number of ounces,
or
$4.95
54.4
ounces = $0.09 per ounce.
PRACTICE TEST 2 ANSWERS AND EXPLANATIONS 161
33. The correct answer is C. To solve, first square
each fraction:
1
2
!
2
+
1
3
!
2
+
1
4
!
2
=
1
4
+
1
9
+
1
16
. Remember that to be added, fractions must
have a common denominator. In this case, since
4 is a factor of 16, the lowest common denomi-
nator is (9)(16) = 144. To convert fractions into
different denominators, you must multiply the top
and bottom of a fraction by the same number. If
1
4
is multiplied by
36
36
, the result is
36
144
. Likewise,
multiplying
1
9
by
16
16
yields
16
144
, and multiplying
1
16
by
9
9
yields
9
144
. Therefore
1
4
+
1
9
+
1
16
=
36
144
+
16
144
+
9
144
=
(36 + 16 + 9)
144
, or
61
144
.
34. The correct answer is G. The easiest way to solve
this problem is to draw a picture similar to the one
below.
Since the route heads straight north from
Hermansville for 120 miles to Jamestown, and then
straight west for 80 miles to Melville, the turn at
Jamestown creates a right angle. If a straight, flat
road existed between Hermansville and Melville, it
would form the hypotenuse of a right triangle with
legs 80 and 120. Using the Pythagorean Theorem
(c
2
= a
2
+ b
2
), you can see that the distance of
this straight route from Hermansville to Melville
would be:
%
(120
2
+ 80
2
)
=
$
(14,400 + 6,400)
=
$
20,800 144
35. The correct answer is B. To solve this problem,
calculate the volume of the aquarium and divide
by 2. Since volume is equivalent to length × width
x height, the volume is 30 × 16 × 12, or 5,760
cubic inches of water. Dividing by two, you see
that half of the tank would be 2,880 cubic inches of
water.
36. The correct answer is K. To solve this problem,
you would multiply the number of possibili-
ties in each officer position. Since the league
selects its 4 officers by first selecting the presi-
dent, then the vice president, then the secretary,
then the treasurer, there are 40 possibilities for
president, 39 possibilities for vice president,
38 possibilities for secretary, and 37 possibil-
ities for treasurer. The total number of dif-
ferent possibilities for the election is therefore
40 × 39 ×38 ×37.
37. The correct answer is A. Because there is a
right angle at S, the point T will lie along the
line through S that is perpendicular to the seg-
ment RS. To solve this problem, find the equa-
tion for the line through S that is perpendicular
to the segment RS and try each answer choice
to find one that lies on the line. Since the line
is perpendicular to segment RS, it will have a
slope that is the opposite reciprocal of the slope
of RS. Since slope is rise/run, the slope of RS
is
(3 2)
(6 2)
=
1
4
. The slope of a line perpen-
dicular to that is 4. Because a point and the
slope of the line are known, the point-slope form
of the equation can be utilized. A line through
point (h, k) with slope m has equation y k =
m(x h). Thus the line through S (6,3) that
is perpendicular to the segment RS has equation
y 3 = 4(x 6). Distributing and adding
like terms, the result is y = −4x + 27. Of
the answer choices, only the point (5,7) falls on
the line.
38. The correct answer is F. To solve the equation
0.2(x 2,700) = x, first distribute:
0.2x 540 = x
540 = 0.8x
675 = x
162 PRACTICE TEST 2 ANSWERS AND EXPLANATIONS
39. The correct answer is A. Given that 0
x 90
and that tan x =
15
8
, x can be pictured in the right
triangle below.
Because tangent is the ratio of the side opposite
the angle to the side adjacent to the angle, the
legs of the right triangle can be labeled as above.
Cosine is the ratio of the side adjacent to the angle
to the hypotenuse, which is not given. It is possi-
ble to eliminate answer choices in such a manner
that it is not necessary to use the Pythagorean
Theorem. Since the side adjacent to x is 8, the
numerator in cos x will be 8, eliminating all but
answers A and E. Since the legs of the trian-
gle are 8 and 15, the hypotenuse will be longer
than either, eliminating answer choice E. Thus
cos x =
8
17
.
40. The correct answer is G. Since the area of the
square pool is given, you must find the area of the
circle, with a radius of 10, and subtract the area
of the pool. The area of a circle is equal to πr
2
,
where r is the radius. The area of this circle is
10
2
π = 100π 314 square feet. Thus the area
of the enclosure is approximately 314 81 = 233
square feet.
41. The correct answer is B. Remember that all par-
allel lines have the same slope, so a line parallel
to y = 2x + 2 will a slope of 2. A quick way
to aid you in solving this problem would be to
eliminate answer choices that do not have slope
2, so answer choices A and E can be immediately
eliminated. Check the point (3,4) in the remain-
ing answer choices. The only choice that works
is y = 2x 2.
42. The correct answer is H. Tangent is the ratio of
the side opposite to the side adjacent to an angle
in a right triangle. Drawing a line that passes
through (3,3) and is perpendicular to the x-axis
creates a right triangle, as shown in the figure
(see below).
Because point (3,3) is given, both legs of the right
triangle have a length of 3. Thus tan ϕ =
3
3
= 1.
43. The correct answer is A. To solve, calculate the
result for each operation and select the smallest
result.
Answer Choice A:
2
3
+ (3) = 2
1
3
Answer Choice B:
2
3
(3) = 3
2
3
Answer Choice C:
2
3
× 3 = 2
Answer Choice D:
2/3
3
=
2
9
Answer Choice E:
[2/3 + (3)]
2
=
7
6
The smallest result is 2
1
3
, which was obtained by
adding.
44. The correct answer is F. To simplify calculations,
you can multiply the entire equation by 12 to obtain
whole numbers and get 4b + 24 = 3. Subtracting
24 from both sides yields 4b = 21. Dividing by 4
yields b =
21
4
, which is a little less than 5. Thus
the correct answer will lie between 4 and 6.
45. The correct answer is B. To find the solution set
for |3a 2| 7, break it up into two separate
inequalities: 3a 2 7 and 3a 2 7. Starting
with 3a 2 7, solving for a yields a 3. With
3a 2 7, solving for a yields a
5
3
. Thus
a is between
5
3
and 3 inclusive.
PRACTICE TEST 2 ANSWERS AND EXPLANATIONS 163
46. The correct answer is K. Tangent is the ratio of
the side opposite to the side adjacent to an angle
in a right triangle. If the distance, in feet, to the
cell phone tower is x, then tan 41
=
200
x
, or x =
200
tan 41
. Since cot 41
=
1
tan 41
, x =
200
tan 41
=
200 cot 41
.
47. The correct answer is A. Since the area of a square
is equal to the square of its sides, multiplying the
sides by
3 will have the effect of multiplying the
area by (
3)
2
= 3.
48. The correct answer is H. In order to solve this
problem, you must realize that since the volume of
a cube is equal to the cube of its sides, multiplying
the length of the sides by
1
2
will have the effect of
multiplying the volume by
1
2
!
3
=
1
8
. The cube
in this problem has a volume of 64, so if you halve
the length of each side, new cube’s volume will be
64
1
8
!
= 8.
49. The correct answer is D. The area of a parallelo-
gram is equal to base × height. In the figure, you
can see that the base of the parallelogram is 7 and
the height of the parallelogram is 9. Thus, the area
of the parallelogram is 9 × 7 = 63.
50. The correct answer is G. In order for 8a
6
b
3
to
be less than zero, either 8 or a
6
or b
3
must be
less than zero. However, it is obvious that 8 > 0
and any number taken to an even power is non-
negative. Thus b
3
<0 and in order for that to be true,
b < 0. Of the answer choices, only b > 0 CANNOT
be true.
51. The correct answer is D. Logarithms are used to
indicate exponents of certain numbers called bases.
By definition, log
a
b = c, if a
c
= b. If log
4
x = 3,
then x = 4
3
, or 64.
52. The correct answer is J. In order for a system of 2
linear equations to have no solutions, the graphs of
the equations must be parallel. Parallel lines have
the same slope. To find the equation whose graph
is parallel to the line in the figure, you must find
the slope of the line between the points (0,4) and
(3,0). Since slope is
rise
run
, the slope is
4
3
. The only
equation with the correct slope of
4
3
is y =
4
3
x +2.
53. The correct answer is C. Of the 80 marbles, only
8 end in 5. If the first marble is drawn and not
replaced, there are 79 marbles left, 7 of which have
a ones digit of 5. Thus the probability that the player
will be a winner is
7
79
.
54. The correct answer is H. To solve this prob-
lem, remember that the formula for slope is equal
(y
2
y
1
)
(x
2
x
1
)
, where (x
1
, y
1
) and (x
2
, y
2
) are two given
points on a line. The equation of the line that passes
through the origin and the point (3,4) will have
slope
(4 0)
(3 0)
=
4
3
. Since the line passes through
the origin, the y-intercept is 0. Thus the correct
equation is y =
4
3
x.
55. The correct answer is C. To solve this problem,
you must remember that in an isosceles triangle,
the base angles have the same measure. Since the
sum of angles is 180
for all triangles, 180 = (a +
30)+2(2a15). Distributing and adding like terms
yields
180 = (a + 30) + 4a 30
180 = 5a
a = 36
Since the base angles are equivalent to 2a 15,
they equal 2(36) 15 = 72 15, or 57
.
56. The correct answer is K. The smallest possible
value will occur when it is negative. A negative
product will result only when one of the numbers
is positive and one is negative. The possible pairs
are then 1 and 6, 2 and 5, 3 and 4, 4 and
3, 5 and 2, and 6 and 1. Of these pairs, the
smallest product is (3)(4) = (4)(3), or 12.
57. The correct answer is A. Start by drawing 3
parallel lines.
Now, try drawing 3 lines in other configurations,
and you will see that there will always be either
164 PRACTICE TEST 2 ANSWERS AND EXPLANATIONS
6 or 7 regions:
Therefore, the correct answer is 4, 6, or 7 distinct
regions, answer choice A.
58. The correct answer is F. Remember that the area
of a parallelogram is equal to base × height. In this
case, the base is [3 (3)] or [2 (4)], both of
which equal 6, and the height is (3 (5)) = 8.
Thus the area is 6 × 8 = 48. The area of tri-
angle QRS,
&
1
2
(b)(h)
'
, is half the area of the
parallelogram, or 24.
59. The correct answer is E. Refer to the following
chart to follow the patter of the sequence.
Since the power of b is one less than the number of
term, the nth term will be ab
n1
. The 643rd term
will then be ab
6431
= ab
642
.
60. The correct answer is H. Since AB is longer than
BC, there are only two possible configurations: B is
between A and C or C is between A and B. In the
case that B is between A and C, AC = AB +BC =
19 + 13 = 32. In the case that C is between A and
B, AC = AB BC = 19 13 = 6. Therefore, AC
can be 6 and 32 only.
PRACTICE TEST 2 ANSWERS AND EXPLANATIONS 165
Reading Test Explanations
PASSAGE I
1. The best answer is A. Jake makes this state-
ment in response to Katherine telling him that she
sometimes has to “throw something together at the
last minute.” This suggests that it does not hap-
pen often and is unintentional. The other answer
choices are not supported by the passage.
2. The best answer is H. During the conversation,
Katherine says, “It’s just that I work two back-to-
back jobs every night,” and later, “Things are just
a little hard for us right now.” You can infer that
Katherine is working hard and barely getting by.
3. The best answer is B. When Jake says,
“Katherine, Maxwell needs to have a healthy
lunch,” he indicates that Max is currently not eat-
ing well. This example has nothing to do with how
well Katherine takes care of her son, but rather
highlights one of the problems she is having in
taking care of her son. The other answer choices
all make reference to her trying in some way.
4. The best answer is J. The word predicament
is used to indicate Jake’s problematic situation
with Max’s lunches. Based on the context of
the paragraph, his situation is somewhat difficult.
Therefore, predicament most nearly means “chal-
lenge,” which refers to a difficult task. The other
answer choices are not supported by the context
of the passage.
5. The best answer is C. Throughout the passage,
Jake is never referred to as either Max’s best friend
or his brother, but Jake does know Max; there-
fore, A, B, and D should be eliminated. He is
noted as being Max’s camp counselor, and has a
good influence on him by encouraging and allow-
ing Max to consume healthier meals. Therefore,
answer choice C is correct.
6. The best answer is F. At the end of the pas-
sage, Jake remarks that the club lasted for two
summers, and then Max moved away. This best
supports answer choice F. Although answer choice
J may appear to be correct because it is implied
that Max enjoyed the meetings of the Sandwich
Club, the author never defines these meeting as
Max’s favorite part of camp.
7. The best answer is D. At several points in the
passage, Katherine refers to how hard it is to find
enough money for everything, and how much she
has to work to make ends meet. This suggests that
Katherine and Max have little money. The other
answer choices are not supported by details in the
passage.
8. The best answer is G. Whenever Jake brings up
problems with Katherine, she seems genuinely
concerned about Max’s well-being. However,
because she works so much and has so many other
things to get done, her attention is often diverted
away from Max. The other answer choices are not
supported by the context of the passage.
9. The best answer is D. When Jake pulled
Katherine aside to discuss the lunch issue, he
indicated that Max’s lunch is a concern of his,
eliminating answer choices A and C. Although
answer choice B may appear to be correct, the
word “incident” implies a one-time occurrence.
Max was consistently bringing unhealthy lunches
to camp, therefore answer choice D, “problem,” is
correct.
10. The best answer is J. The passage states that
Jake thought of the Sandwich Club because he
needed to come up with a solution to help Max
eat healthier lunches. The last paragraph also men-
tions the fact that once Max moved away, the
Sandwich Club ended. These facts best support
answer choice J. The other answer choices are
beyond the scope of the passage.
PASSAGE II
11. The best answer is B. The passage states that
“None but the fearless and inventive, the most
resourceful and curious, would dare to undertake
such a venture,” which clearly suggests that this
was a challenge. The passage later goes on to say
that “All of the wonders of those states in the
West are, in part, the result of this expedition.”
This implies that western expansion was impor-
tant. Answer choice A may appear to be correct;
however, the passages tells us that fur traders and
trappers had traversed the wilderness to reach the
west prior to Lewis and Clark’s journey.
12. The best answer is F. The passage discusses
many aspects of the Lewis and Clark expedition,
and mentions several characteristic of the country,
describing to the reader that the expedition led to
“the unknown lands west of the Mississippi” and
that “The path they were to carve out would be the
first of its kind.” Answer choice H may appear to
be correct, however, the passage mentions that
the men on the expedition were volunteers for
“North Western Discovery,” not the “Northwest
Passage.”
166 PRACTICE TEST 2 ANSWERS AND EXPLANATIONS
13. The best answer is D. The passage states that,
“The settlers who came after Lewis and Clark
went forward with blind-devotion knowing then
that it could be done.” This suggests that the Expe-
dition gave other people confidence that they, too,
could cross the United States because they knew
that it had already been done by other travelers.
Answer choice A is incorrect because the pas-
sage explicitly states that after Lewis and Clark’s
historic journey, many people traveled by land.
Answer choices B and C are beyond the scope of
the passage.
14. The best answer is H. The passage explicitly
states that the Lewis and Clark expedition was
intended to “find the fastest water route across
North America.” In addition, the passage states
that previous to this journey, “virtually no one
had attempted to cross the stretch of land between
the mighty Mississippi and the vast Pacific Ocean
using only water routes.” Although Lewis and
Clark did catalogue new species of animals on
their journey—answer choice G—this was not
their primary objective in traveling across the
United States.
15. The best answer is B. The second paragraph
states that, “Months later, in May, the party gath-
ered in St. Lewis. The forty some men were to
travel . . . Answer choice B best fits the context
of the paragraph. The previous statement does not
support the idea that party means “a celebration,”
“a segment of the population,” or a “meeting to
discuss business,” so answer choices A, C, and D
can be eliminated.
16. The best answer is G. In the third paragraph, the
men on the expedition are told to have written
of their “health and high spirits.” The statement
is also made that the men were “all eager to
explore,” in spite of the potential dangers they
faced in their long journey. This best supports
answer choice G.
17. The best answer is C. It is reasonable to infer that
the word “forge,” as it is used in the passage “the
travelers continued to forge west in search of an
efficient trade route using only the rivers”–refers
to advancing on a path or journey, answer choice
C. The other answer choices do not make sense
within the context of the passage.
18. The best answer is J. The fourth paragraph men-
tions that “the Sioux and the Blackfeet tried
to impede the group’s progress on more than
one occasion.” This suggests that the Expedition
would likely face danger in the form of conflict
with indigenous people, answer choice J. The
other answer choices are not supported by details
in the passage.
19. The best answer is D. As stated in the passage,
the men set out on three boats, “all of which
were moved by sails, towropes, poles, or oars.”
These methods involve man power, wind power,
or rowing power. Although the passage indicates
that steam power eventually replaced the boats,
nowhere does it mention that steam power was
used on the Expedition.
20. The best answer is F. According to the passage,
“After receiving wilderness training in Washing-
ton D.C., Meriwether Lewis set out . . . This
indicates that Lewis was the only member of the
Expedition to have received wilderness training.
All of the other answer choices are mentioned
explicitly in the passage as having been done by
all members of the Expedition.
PASSAGE III
21. The best answer is D. Although the passage
discusses Porters renowned ability to candidly
address artists’ works, illustrates the influence
that several famous artists had on his works, and
assesses his unusual methods of painting and cri-
tiquing artwork, none of these are the main focus
of the passage. Therefore, answer choices A, B,
and C can be eliminated. The passage gives a brief
summary of Porters life and discusses all of the
above topics as points that shaped his career. This
best supports answer choice D.
22. The best answer is H. At the end of the passage,
the author states that it is sad that Porter “is still
virtually unknown outside of art circles,” and that
“This remarkably insightful, articulate, creative
individual needs to be discovered by the common
man and revered for his continuing influence on
the artists of today.” Clearly, the author thinks
very highly of Porter and his works and believes
that he deserves to be honored (“revered”) for
his influence on today’s artists. These details
best support answer choice H. Answer choice
F is incorrect because the author is clearly not
“detached,” or indifferent. Answer choice G is
incorrect because the author is not merely “tol-
erant,” or just able to withstand Porter. Answer
choice J is incorrect because the author obviously
does not “abhor,” or hate, Porter and his works.
23. The best answer is A. As stated in the passage,
“what made Porter so famous was his knack for
PRACTICE TEST 2 ANSWERS AND EXPLANATIONS 167
responding directly to an artist’s work.” Answer
choices B and D are incorrect because the pas-
sage states that those are the things that Porter
did NOT do, for he found criticisms based on
those criteria to be insignificant and meaningless.
Answer choice C is beyond the scope of the pas-
sage; Bernard Berenson’s influence on Porters art
critiques is not discussed.
24. The best answer is H. Porters personal philoso-
phy regarding his paintings was that they should
be “personal, emotional, and representative of its
subject, while at the same time be boldly col-
orful, expressive, and generally abstract.” This
description of his works supports all of the answer
choices except “trite,” answer choice H, which
means “ordinary or dull.”
25. The best answer is B. The first paragraph of the
passage simply tells how the author first came
to meet Fairfield Porter, and provides a smooth
transition into the life of Fairfield Porter. Answer
choice A may appear to be correct; however, the
author simply tells the reader how he came across
Porter; there is no overview of what is to come
in the passage. Answer choices C and D are not
supported by information found in the passage.
26. The best answer is H. The passage states that,
“Fairfield Porter, despite being remarkably intel-
ligent, appeared to be lacking any natural artistic
talents.” Therefore, his high intelligence level was
not correlated at all to his level of artistic ability.
This best supports answer choice H. Porters intel-
ligence is not discussed in terms of the criteria
listed in answer choices F, G, and J.
27. The best answer is D. The author states that,
“This remarkably insightful, articulate, creative
individual needs to be discovered by the com-
mon man and revered for his continuing influence
on artists today.” The author is clearly praising
Porter and his continuing influence. It makes the
most sense that the author believes that Porter
should be “honored” for his continuing influence
on artists today. The other answer choices do not
make sense in the context of the paragraph.
28. The best answer is J. It is stated in the passage
that Porter was primarily an art critic until “1961,
when he decided to pursue a full time painting
career.” This was approximately 30 years after he
returned from Italy, as he spent time between the
years 1931 and 1932 in Italy learning to appreciate
and critique artworks. The other answer choices
are not supported by details in the passage.
29. The best answer is A. As stated in the pas-
sage, “Between the years 1931 and 1932, Fairfield
spent the majority of his time in Italy learning
to appreciate and critique the works of the great
Renaissance painters. His training came from
both . . . This suggests that Porter was continu-
ing his training as an art critic, answer choice A.
The other answer choices are outside the scope of
the passage.
30. The best answer is G. The passage preceding this
phrase discusses Porters fame as an acclaimed art
critic. The passage then goes on to state that, “The
other side of his fame, his uncommon approach
to painting, is just as important to an understand-
ing of Fairfield Porters contributions to the world
of art.” This contrast indicates that Porter was
famous both for his criticisms of art, as well as his
artwork itself–answer choice G. Answer choice F
may appear to be correct, however, the passage
does not ever define Porters level of fame as an
artist versus his level of fame as an art critic.
PASSAGE IV
31. The best answer is A. The passage states that,
“without dark matter, there are many cosmolog-
ical phenomena that are difficult to explain.” In
the context of the sentence, it would make the
most sense that without dark matter, there would
be many “occurrences” or incidents that would be
difficult to explain. Answer choice B may appear
to be correct; however, the passage does not indi-
cate the nature of the phenomena and whether or
not they are problematic.
32. The best answer is H. The passage states that,
“Dark matter . . . is not readily observable because
it does not refract light or energy directly. Its exis-
tence can only be deduced because of the effect
that it has on surrounding matter. In fact, some
members of the scientific community have argued
that dark matter does not actually exist.” This best
supports answer choice H. Answer choice F is
incorrect because the evidence for the existence of
dark matter is its effect on surrounding matter; this
discussion of dark matters effect on surrounding
matter also eliminates answer choice J. Answer
choice G is not supported by details in the passage.
33. The best answer is C. As stated in the passage,
“It has been asserted that not only does dark mat-
ter exist, it may also be responsible for the Milky
Way’s unusual shape.” The passage then goes on
to discuss the way in which dark matter probably
affects the shape of the Milky Way. Answer choice
168 PRACTICE TEST 2 ANSWERS AND EXPLANATIONS
B may appear to be correct; however, the passage
does not indicate that the Magellanic clouds have
enormous mass. In fact, the passage explicitly
states that “The interaction referenced involves
two smaller galaxies near the Milky Way, called
Magellanic clouds, moving through an enormous
amount of dark matter.” Answer choices A and D
are not supported by details in the passage.
34. The best answer is F. As stated in reference
to Romanowsky’s theory, “They point to the
existence of several elliptical shaped galaxies sur-
rounded by very little dark matter as evidence that
dark matter is not, in fact, the cause of the warped
galaxies.” By showing galaxies that are similar in
shape to the Milky Way but NOT surrounded by
enormous amounts of dark matter, the theory illus-
trates that dark matter may not affect the shape and
formation of galaxies, answer choice F. Answer
choice G may appear to be correct; however, the
passage explicitly states that Romanowsky’s the-
ory is not intended “to conclude that dark matter
does not exist.”
35. The best answer is A. The last paragraph states
that “the movement of [the Magellanic] clouds
through the dark matter seems to create a wake
that enhances their gravitational influence on the
Milky Way.” Earlier in the paragraph, when dis-
cussing the Magellanic clouds, the passage states
that when a cloud moves through dark matter,
it “enhances the gravitational pull that the two
Magellanic clouds could have on the Milky Way
and other surrounding bodies.” This best supports
answer choice A. The other answer choices are
not supported by details found in the passage.
36. The best answer is J. The third paragraph dis-
cusses the substance of Newton’s hypothesis and
the consequences that his hypothesis had on
the existence of dark matter. When applying
this hypothesis, it seems that dark matter must
exist: “something that is not easily observed must
be exerting the necessary force to create the
warped shape of the galaxy.” This best supports
answer choice J. Answer choice F may appear
to be correct; however, it is simply stated that
Newtonian physics provide the strongest evidence
for dark matter. It is not stated anywhere in the
passage how dark matter was first discovered.
37. The best answer is D. The passage states that
“they apparently believe that the results of their
studies cast doubt on some of the conventional
theories of galaxy formation and manipulation.”
Answer choices A and C do not make sense in
the context of the sentence, because if a theory
was “easily understood” or “strictly interpreted,”
there would not be a lot of room for doubt to be
cast upon the theory. Answer choice B is incor-
rect because if a theory was “formally disputed,”
opposing viewpoints would already exist on that
theory. That the result of these studies cast doubt
on some “generally accepted” theories makes the
most sense; therefore, answer choice D is correct.
38. The best answer is H. There is nothing in the
passage to indicate with certainty that dark matter
has no influence on surrounding celestial bod-
ies. Although Aaron Romanowsky suggests that
dark matter is not responsible for warped galax-
ies, there is no discussion in the passage to show
that he believes dark matter has no influence
whatsoever on galaxy shape or other cosmolog-
ical phenomena. The other answer choices are
all mentioned in the passage as scientific theories
regarding dark matter.
39. The best answer is C. As stated in the passage,
dark matter is “a substance that is not readily
observable because it does not refract light or
energy directly.” This best supports answer choice
C. The other answer choices are not supported by
details found in the passage.
40. The best answer is F. The passage describes
dark matter as surrounding and impacting galaxies
composed of common matter, such as the Milky
Way. This best supports answer choice F. Answer
choice G is incorrect because whether dark matter
truly exists is still a topic of debate among scien-
tists. Answer choice H is incorrect because the
passage does not provide the magnitude of the
amount of dark matter in the universe. Answer
choice J is incorrect because dark matter is not
directly observable at all; its effect on galaxies is
the only proof of its existence.
PRACTICE TEST 2 ANSWERS AND EXPLANATIONS 169
Science Reasoning Test Explanations
PASSAGE I
1. The best answer is C. Table 1 provides informa-
tion on how many pinecones were produced at
each of the six sites. When looking at the A4 and
A5 column, the only sites at which A5 trees pro-
duced more pinecones per tree than A4 trees were
S1, S2, S4, and S5. This best supports answer
choice C.
2. The best answer is H. The results of Experi-
ment 1 are shown in Table 1. According to these
results, the only site at which A1 trees produced
more pinecones than the other trees was at S4,
answer choice H.
3. The best answer is C. According to Table 2, only
S4 trees produced fewer pinecones than did the
trees at S6. This suggests that A1 trees produced
more pinecones at S6 than at S4, because A1 trees
were planted with A2, A3, A4, and A5 trees in
both experiments.
4. The best answer is G. Table 2 provides informa-
tion on how many pinecones were produced per
A1 tree at each of the six sites. To answer the
question, you must look at which A1 tree pro-
duced the most pinecones. The A1 trees that were
planted with seedlings from A2 at S3 produced 9.6
pinecones per tree, more than any other tree on
the table. This information best supports answer
choice G.
5. The best answer is A. To answer this question,
you must read the procedures for both Experi-
ment 1 and Experiment 2. In Experiment 2, “150
containers were prepared with 5 A1 seedlings
and 5 seedlings from either A2, A3, A4, or
A5.” This says that, unlike Experiment 1, trees
from more than one population were combined
in Experiment 2. Therefore, answer choice A is
correct.
6. The best answer is H. The procedure for Exper-
iment 2 states, “150 containers were prepared
with 5 A1 seedlings and 5 seedlings from either
A2, A3, A4, or A5.” Therefore, 10 seedlings
were planted in each container. This data supports
answer choice H.
PASSAGE II
7. The best answer is C. The point at hr =16 for the
“Ingredient A extended-release” line is at about 15
on the vertical axis. The point at hr = 16 for the
“Ingredient B extended-release” line is at about 4
on the vertical axis. Therefore, the difference is
about 11.
8. The best answer is H. This question can be
answered by observing the trends in the Figures or
simply thinking critically about how medication
generally works. According to Figure 2, it takes
a bit of time for the medication to start work-
ing, the symptoms decline for a period of time,
then, presumably once the medication starts to
wear off, the symptoms return. Figure 1 shows
that symptom relief is inversely proportional to
concentration of the medication in the body.
9. The best answer is A. According to Figure 1, the
concentration of immediate-release Ingredient A
increases most immediately following taking the
pill. This is evident by the extended steep upward
trend of the line.
10. The best answer is F. Table 1 shows that nearly
as many subjects given the placebo (5%) reported
feelings of weakness as did subjects given the
drug (6%). The difference of only 1% shows that
there is a high level of uncertainty over whether
the drug actually caused the feelings of weakness.
11. The best answer is D. Figure 2 reflects the mean
(average) symptom score of all the subjects given
the drug. To find the 8-hour period of the graph
where symptoms scores changed the least, find
the 8-hour period of the curve over which slope
changes the least. This occurs roughly between
hr = 14 and hr = 22.
PASSAGE III
12. The best answer is G. By looking at Figure 1,
you can determine the range in pressure at which
each atmospheric layer can exist. Beginning with
answer choice F, calculate the difference between
the lowest pressure and the highest pressure at
which the atmospheric layer can exist:
Mesosphere: 14.5 psi 7.5 psi = 7 psi
Thermosphere: 17.8 psi 10.3 psi = 7.5 psi
Stratosphere: 9.5 psi 3.8 psi = 5.7 psi
Troposphere: 5.0 psi 1.0 psi = 4.0 psi
The pressure in the thermosphere has the greatest
range, answer choice G.
13. The best answer is A. To answer this question
you need to look at the information on cumulus
clouds in Figure 1. Figure 1 suggests that cumulus
clouds form in the mesosphere. The mesosphere’s
170 PRACTICE TEST 2 ANSWERS AND EXPLANATIONS
pressure ranges from 8 psi to 12 psi, and the
average temperature for this range is 35
C. This
information supports answer choice A.
14. The best answer is G. The key in Figure 1
includes two lines: temperature and pressure. As
the pressure line is increasing (positive slope), the
temperature line is decreasing (negative slope).
This suggests that as pressure within the atmo-
spheric layer increases, temperature decreases
only, answer choice G.
15. The best answer is B. Answering this question
requires you to look carefully at the range of
temperature and pressure combinations at which
clouds in the stratosphere form, as shown in
Figure 1. Because the clouds here begin to form
under 30
C and most are located below a pressure
of 10 psi, this combination is correct. The other
choices designate ranges of the atmosphere either
above or below the stratosphere.
16. The best answer is J. At a pressure of 7 psi,
Figure 1 shows that cloud formation will most
likely occur in the stratosphere. According to
Figure 1, clouds that form in the stratosphere
are stratus clouds. This information best supports
answer choice J.
PASSAGE IV
17. The best answer is D. Figure 1 shows the per-
cent of fish that survive after exposure to water
with bacteria present. The fish species with the
highest percent (85%) of fish that survived after
prolonged exposure to bacteria (light gray bar)
was fish species E. Answer choice D is correct.
18. The best answer is G. To answer this ques-
tion, you must look at the difference between the
light gray and dark gray bars for each species
in Figure 1. Answer choice J can be eliminated
because the percent of fish that survived with and
without exposure to bacteria are almost identical.
Species C has the greatest difference in percent
between the two, answer choice G.
19. The best answer is B. According to Table 1,
fish that live in deep water without plants have
a moderate ability to combat bacteria.
20. The best answer is G. By looking at Table 1, you
can see that as you go down the column, the expo-
sure to waterborne bacteria increase. Also as you
go down the column, the relative ability to com-
bat bacteria increases. The information supports
answer choice G.
21. The best answer is B. According to Figure 1,
Species A has the lowest percent of fish surviv-
ing to adulthood. Table 1 indicates that Species A
fish live in shallow water with plants. Therefore,
answer choice B is correct.
PASSAGE V
22. The best answer is H. This question asks you
to identify the assumption that Paleontologist
1 must have made while discussing the find-
ing of the fossilized bones. Answer choice F
can be eliminated because the paleontologist
refers to the velociraptor as a known dinosaur.
Paleontologist 1 says, “The form and function
of the velociraptor has been misunderstood until
this important discovery.” By stating that the
dinosaur has been misunderstood, Paleontolo-
gist 1 is saying that it has been mischaracterized
until now. This statement best supports answer
choice H.
23. The best answer is C. Paleontologist 1 states,
“You can see that there are cuts within the
arm/wing bones of this dinosaur, indicating that
it was caught while in flight.” The paleontologist,
then goes on to say, “Perhaps it was attempting
an escape from a more predatory dinosaur.” This
suggests that while trying to escape from a larger
predator, the predator caught the velociraptor,
answer choice C.
24. The best answer is G. According to Paleontol-
ogist 2, “long arm bones are indicative of the
dinosaurs ability to scavenge prey and fend off
larger predators.” It makes sense that Paleontol-
ogist 2 would conclude that, because the new
dinosaur species had long arm bones, it could be
a scavenger.
25. The best answer is C. According to Paleontol-
ogist 2, “The long arm bones are indicative of
the dinosaurs ability to scavenge prey and fend
off larger predators.” This implies that the reason
that the velociraptor had long arm bones was to
scavenge for food.
26. The best answer is F. This question asks you
to summarize the main idea of Paleontologist
1’s viewpoint. Answer choices G and H can be
eliminated because only Paleontologist 2 supports
them. Paleontologist 1 says that “the velociraptor
was definitely capable of flight” and “the veloci-
raptor has been misunderstood until this important
discovery.” These two statements support answer
choice F.
PRACTICE TEST 2 ANSWERS AND EXPLANATIONS 171
27. The best answer is A. Both Paleontologists men-
tion the Tyrannosaurus rex as a possible threat
either to the velociraptors food source or to its
very life. This best supports answer choice A.
Answer choices B, C, and D do not support the
views of both paleontologists, and therefore, can
be eliminated.
28. The best answer is G. By reading the viewpoints
of both Paleontologist 1 and Paleontologist 2, you
can determine what they thought of the discov-
ery of the velociraptor bones. Answer choices
H and J can be eliminated because they are not
supported by either passage. Both paleontologists
extracted useful information from the discovery
of the bones. Therefore, answer choice G is the
best answer.
PASSAGE VI
29. The best answer is B. Figure 1 provides informa-
tion on the compositions of mountain peaks. At a
distance of 9 km along the mountain range, the
peak composition is shown in the key as shale.
This supports answer choice B.
30. The best answer is F. To determine the cor-
rect answer, you must look at the key in Figure
1, which shows the different composition of the
mountain peaks. It defines limestone as “particles
with diameters under 0.5 mm,” shale as “com-
posed mostly of particles with diameters under
175 mm,” and slate as, “composed mostly of par-
ticles with diameters over 175 mm.” The only
difference between the different compositions is
the size of the particle, answer choice F.
31. The best answer is D. As the peak section
heights in Table 1 increase, the percentage of
the year that peak section is exposed to wind
also increases. Since a height of 5.5 m to 6.0 m
is higher than the other values in the table,
the percentage of the year that the peak section
would be exposed to the wind should also be
greater than the other values in the table. The
only answer choice with a value greater than
the other percentages in the table is answer
choice D.
32. The best answer is F. Figure 2 shows that Peak
D is exposed to wind erosion for a greater per-
centage of the year than Peak C. Table 1 suggests
that the percentage of the peak exposed to wind
is directly proportional to peak section height.
Therefore, because Peak D is exposed to the
wind for longer than Peak C is, Peak D must be
taller than Peak C. This information best supports
answer choice F.
33. The best answer is C. Table 1 shows that
as the peak section height increases by equal
increments, the percentage of the year that
peak section is exposed to wind also increases
by approximately the same amount. Therefore,
the slope of the graph is positive and the
graph is a straight line as shown in answer
choice C.
34. The best answer is G. The passage states
that “the peaks of mountains often lose sed-
iment due to wind erosion.” By losing sed-
iment, the mountain peaks are losing mass.
This supports answer choice G. Answer choice
J can be eliminated because the information
is irrelevant to the data presented in the pas-
sage.
PASSAGE VII
35. The best answer is A. According to the informa-
tion in Table 2 and Table 3, the bacteria grew
quicker when left under a lamp. This suggests
that bacteria grow faster in warm environments,
answer choice A. Answer choice C can be elim-
inated because it is irrelevant to the information
presented in the passage.
36. The best answer is G. Table 3 provides infor-
mation on the proportional growth of bacteria in
four different environments. The moist side of
Dish 2 produced 1.15 times the amount of bac-
teria in the control sample, which is more than
the other three environments produced when com-
pared to the control sample. This best supports
answer choice G.
37. The best answer is C. Table 1 provides the results
of Experiment 1. Answer choices A and D can be
eliminated because they are not supported by the
data. For both dishes, the bacteria growth was
greater on the moist side. This information best
supports answer choice C.
38. The best answer is H. More than one sample
of bacteria was put into each Petri dish. To avoid
skewed results, only one sample of bacteria should
be placed in each Petri dish. Answer choice H is
the best answer. Answer choice J can be elim-
inated because varying the size of the starting
sample would not alter the results.
39. The best answer is D. Both Table 2 and Table 3
show that the bacteria growth rate is highest in
172 PRACTICE TEST 2 ANSWERS AND EXPLANATIONS
moist, warm places. The only answer choice that
is both warm and moist is beneath a rock in a
tropical forest. The other answer choices can be
eliminated because they contain only one or none
of the two conditions.
40. The best answer is J. According to the
question, “bacteria double population size in
short intervals.” The only answer choice that
includes recording data in short intervals is
answer choice J.
PRACTICE TEST 2 ANSWERS AND EXPLANATIONS 173
Writing Test Explanation
Because grading the essay is subjective, we’ve chosen
not to include any “graded” essays here. Your best bet
is to have someone you trust, such as your personal
tutor, read your essays and give you an honest critique.
If you plan on grading your own essays, review the
grading criteria and be as honest as possible regarding
the structure, development, organization, technique, and
appropriateness of your writing. Focus on your weak
areas and continue to practice in order to improve your
writing skills.